LEADERSHIP AND MANAGEMENT Flashcards

1
Q

A five-year-old has been hospitalized for 24 hours. He is on skeletal traction for the treatment of a right femur fracture. You walk into the room and find him crying. His right foot is pale, and you feel no pulse. What is your priority nursing intervention?

A. Reassess the foot in twenty minutes.

B. Readjust the traction.

C. Administer the ordered as needed pain medication.

D. Notify the physician.

A

Explanation

Choice D is correct. The assessment findings indicate circulatory compromise to the right foot. This may be secondary to arterial injury distal to the fracture or compartment syndrome. It is an emergency and the nurse should notify the physician immediately to obtain appropriate orders for evaluation and intervention.

Choice A is incorrect. Although reassessment is important, any sign of circulatory compromise should be addressed immediately.

Choice B is incorrect. While readjustment of traction may be necessary, notifying the physician regarding the signs of circulatory impairment is of utmost importance. Physicians may decide on appropriate further interventions.

Choice C is incorrect. The nurse should give analgesics to address the child’s pain. However, the administration of pain medication will not resolve the issue of circulatory impairment and it is not the priority nursing action that should be taken.
NCSBN Client Need
Topic: Physiological Integrity;Subtopic: Physiological Adaptation
Reference:
Kozier and Erb’s Fundamentals of Nursing; Health assessment; musculoskeletal impairment.

How well did you know this?
1
Not at all
2
3
4
5
Perfectly
2
Q

Which of the following is an improper technique for correcting written documentation? Select All That Apply.

A. Draw a line through the error, write the date, time, and reason for the error, and add your initials

B. Use correction tape and write over the error so there is no confusion

C. Write over the error in darker ink

D. Completely black out the error with a black marker

A

Explanation

Choices B, C, and D are correct. All of these practices are inappropriate methods of correcting written documentation. Using a tape, writing over the sentence using a black ink, and blacking out using black marker are attempts to conceal the original documentation and may be considered illegal in a court. In a court of law, the court needs to see the underlying data that were corrected. No effort should be made to obliterate the error.

Choice A is incorrect. It is not illegal for medical professionals to make the necessary updates to records, as long as they follow proper methods and do not obscure information. Choice A, in fact, is the correct technique for correcting the written documentation.

NCSBN Client Need I Topic: Health Promotion and Maintenance

Reference: Nursing Health Assessment: A Best Practice Approach (Wolters/Klewer); Chapter 4: Documentation and Interprofessional Communication; Lesson: Accuracy and Completeness

How well did you know this?
1
Not at all
2
3
4
5
Perfectly
3
Q

The nurse is caring for a client who has the following clinical data as shown in the Exhibit. Which action should the nurse take based on the 14:00 vital signs?Click the Exhibit button for additional information.

A. Reassess the client’s vital signs (VS) in one-hour.

B. Notify the primary healthcare physician (PHCP).

C. Perform a physical assessment.

D. Stop the infusion of prescribed D5W

A

Explanation

Correct Answer is B. At 11:00, the client was given intravenous haloperidol, which subsequently increased the client’s pulse and temperature. These changes reflect neuroleptic malignant syndrome (NMS). NMS is brought upon by atypical and typical antipsychotics and is manifested by muscle rigidity, tachycardia, and a fever. The nurse needs to report the 14:00 findings to the provider because of hypertension, progressive tachycardia, and fever.

Choice A is incorrect. The client is showing signs of NMS. Priority action is to contact PHCP. The monitoring of the client can follow later.

Choice C is incorrect. Performing a physical assessment is not necessary at this time as the nurse has sufficient reason to notify the provider.

Choice D is incorrect. Priority action is to report to the PHCP. Stopping the D5W infusion is not immediately necessary.

How well did you know this?
1
Not at all
2
3
4
5
Perfectly
4
Q

The nurse is caring for a patient following the placement of a gastrostomy tube. The Unlicensed Assistive Personnel (UAP) reports thin, pale, and yellow-green drainage with sour odor and a small amount of blood. Which is the best action for the nurse to take?

A. Obtain specimen for culture.

B. Assess the drainage.

C. Instruct UAP to obtain full set of patient’s vitals.

D. Assess patient’s temperature for fever.

A

Explanation

B is correct. The nurse should assess the patient’s drainage to confirm it is within the reasonable expectations for the patient’s condition. Up to 1500mL/day of thin, pale, yellow-green drainage with sour odor and a small amount of blood would be expected for this patient.

A is incorrect. There would be no reason to culture this drainage since it is within expectations for the patient’s condition.

C is incorrect. This would not be an indication to collect a unique set of vitals since this drainage is expected with the placement of a gastrostomy tube. If there is any doubt, the nurse should visualize and assess the patient, not delegate this task to the UAP.

D is incorrect. There would be no reason to expect the patient would be febrile since this drainage is usually scheduled with the gastrostomy tube.

NCSBN Client Need:
Topic: Management of care; Sub-Topic: Assignment/Delegation

Reference: (Lewis, Dirksen, Heitkemper, Bucher, & Camera, 2011, p. 378)

How well did you know this?
1
Not at all
2
3
4
5
Perfectly
5
Q

The primary objective of identification of similarities and differences among the cultural beliefs of the patients by the nurse is to:

A. Communicate with the family

B. Make sure the proper diet is ordered

C. Perform a spiritual consult

D. Avoid making assumptions

A

Explanation

Choice D is correct. Making assumptions or generalizations about a patient’s spiritual needs based on ethnic or religious affiliation is almost sure to be an oversimplification. The nurse should be able to identify similarities and differences among the cultural beliefs of the patients. Just because a patient belongs to certain culture or ethnicity, it is incorrect to generalize their spiritual needs.

Choices A, B, and C are incorrect. Ordering a specific diet as per the patient’s specific cultural or religious preference is certainly warranted. However, generalizations can not be made here either and knowing patient’s specific preference will help the nurse cater to patient’s dietary or spiritual needs. Communicating with the family and performing a spiritual consult should also be done at the patient’s request. While identification of cultural similarities and differences among the patients can help guide these processes, these are not the primary objectives. The primary objective is to avoid making assumptions.

NCSBN Client Need: Topic: Psychosocial Integrity

Resource: Nursing Health Assessment: A Best Practice Approach (Wolters/Klewer); Chapter 10: Cultural Assessment; Lesson: Characteristics of Culture

How well did you know this?
1
Not at all
2
3
4
5
Perfectly
6
Q

What tool, or graphic display, that is shown in the Exhibit can assist the nurse in understanding the interrelationships of the family with the environment, as consistent with systems theory?

A. Histogram

B. A Scatter-gram

C. Genogram

D. Ecomap

A

Explanation

The Correct Answer is D.The tool, or graphic display that is shown above is an ecomap or an ecogram. Ecomaps can assist the nurse in understanding the interrelationships of the family with the environment, as consistent with systems theory Ecomaps show the interrelationships of individuals, families, and communities with their external environment and the forces and relationships that impact on the individual, family, and community.

Histograms and scattergrams show statistical data and not the interrelationships of individuals, families, and communities with their external environment and the forces and relationships that impact on the individual, family, and community.

Genograms show medical information and risk factors in a realistic manner and not the interrelationships of individuals, families, and communities with their external environment and the forces and relationships that impact on the individual, family, and community.

Choice A is incorrect. Histograms show statistical data and not the interrelationships of individuals, families, and communities with their external environment and the forces and relationships that impact the individual, family, and community.

Choice B is incorrect. Scatter grams show statistical data and not the interrelationships of individuals, families, and communities with their external environment and the forces and relationships that impact on the individual, family, and community.

Choice C is incorrect. Genograms show medical information and risk factors in a realistic manner and not the interrelationships of individuals, families, and communities with their external environment and the forces and relationships that impact on the individual, family, and community.

How well did you know this?
1
Not at all
2
3
4
5
Perfectly
7
Q

A 30-year old patient presents to the Emergency Department with alcohol withdrawal seizures. The psychiatry nurse understands that the patient will soon be admitted to the non-medical psychiatric care unit. To keep this patient safe, the nurse must perform which priority nursing action?

A. Ask the physician for clonazepam prescription, an anxiolytic that may help with the withdrawal symptoms.

B. Ensure that a working IV pump is set up at the patient’s bedside.

C. Order a STAT arterial blood gas.

D. Pad the side rails of the patient’s assigned bed.

A

Explanation

Choice D is correct. The patient presented with alcohol withdrawal seizures. The priority nursing action is to pad the bed’s side rails to prevent injury since the patient is at high risk of a recurrent seizure. In an acute care setting, side rails are often used as a medical assistive device and not a restraint. Side rails are considered a restraint only if the intent is to prevent the patient’s free access and keep them in bed. In the setting of seizure precautions, side rails are raised, and the bumper pads are used as a medical assistive device.

Choice A is incorrect. While clonazepam may help with the anxiety associated with alcohol withdrawal, it is not the drug of choice in managing alcohol withdrawal. Instead, diazepam, lorazepam, and chlordiazepoxide are used most frequently to treat or prevent alcohol withdrawal. Furthermore, providing the patient with this medication is not the priority action in patient safety.

Choice B is incorrect. Since the patient will be admitted to a non-medical psychiatry floor, continuous intravenous infusion is not permitted while on that unit. However, necessary injections, oral medications, or other non-invasive procedures performed while on the non-medical unit.

Choice C is incorrect. Ordering a STAT arterial blood gas is not necessary when the patient arrives at the psychiatry unit. Before the patients are sent to the non-medical psychiatry floor, they are already deemed clinically stable and medically cleared.

NCSBN client need | Topic: Safety and Infection Control, Injury Prevention

Reference: Volpicelli, MD, Ph.D., Teitelbaum, MD S. Management of moderate and severe alcohol withdrawal syndromes.

How well did you know this?
1
Not at all
2
3
4
5
Perfectly
8
Q

During a holiday party in your long-term care facility, you take unplanned and surprise candid photographs of residents enjoying the festivities for the monthly newsletter. What have you done?

A. You have appropriately facilitated reminiscence therapy with photographs.

B. You have violated federal law.

C. You have effectively facilitated group cohesiveness with these photographs.

D. You have violated state law.

A

Explanation

The correct answer is B. You have violated the federal law entitled the Health Insurance Portability and Accountability Act of the federal government. The Health Insurance Portability and Accountability Act mandates confidentiality and protects the clients’ right to confidentiality. Taking photographs without resident consent is a violation of the federal Health Insurance Portability and Accountability Act.

Choice A is incorrect. You have not appropriately facilitated reminiscence therapy with these photographs. These surprise group photographs are NOT appropriate.

Choice C is incorrect. You have not appropriately facilitated group cohesiveness with these photographs. These surprise group photographs are NOT appropriate.

Choice D is incorrect. You have not violated state law, but you have violated federal law. Health Insurance Portability and Accountability Act is governed by federal law.

Reference: Berman, Audrey, Snyder, Shirlee, and Geralyn Frandsen. Kozier and Erb’s Fundamentals of Nursing: Concepts, Process, and Practice.

How well did you know this?
1
Not at all
2
3
4
5
Perfectly
9
Q

Due to an absent staff nurse, the postpartum unit is assigned a nurse from the “medical” ward as a floater. Which of the following patients should the charge nurse assign to the float nurse?

A. A 20-hour postpartum client who will be discharged the following morning.

B. A 16-hour postpartum client who had eclampsia during delivery.

C. A 10-hour postpartum client who has soaked 4 perineal pads in one hour.

D. A 5-hour postpartum client whose fundus is still not at the midline.

A

Explanation

Correct Answer is B. This client can be assigned to the float nurse. The nurse is floating from the medical unit to the postpartum unit. Eclampsia is a complication of preeclampsia and is characterized by high blood pressure and seizures. This client remains at risk for a seizure. The goals of management of Eclampsia involve controlling seizures and controlling hypertension. Medical unit nurses understand and are experienced in taking care of clients having a seizure.

“Float Nurse” refers to someone who is permanently assigned to a specific unit but is asked to cover another group because of staffing needs temporarily. Some hospitals have
“float pools” where the nurses float from one to another, based on staffing needs (sick calls, increased census, etc.). Nurses in “float pools” are well versed with a variety of scenarios and are often able to handle many assignments with ease. However, their experience tends to be “generalized” rather than “specialized.” To adequately use the strengths of a “float” nurse, appropriate patient assignments should be made.

A charge nurse should consider a few issues before making patient assignments to a float nurse. More general diagnoses are better suited for the float nurses because such determinations have more standardized plans of care. Regarding unit-specific patients, the float nurses may not have specialized experience to care for such specific diagnoses. These unit-specific patients are better assigned to experienced nurses who are trained for those particular aspects of the patients’ diagnoses and who are more familiar with those complicated care plans.

Choice A is incorrect. The client is being discharged from the postpartum unit. She needs to be assessed whether she has the capacity to take care of her baby once at home. She also needs to be educated by the nurse about newborn care. A specialized nurse with postpartum unit-specific experience should be assigned to this client.

Choice C is incorrect. This client seems to be having primary Post-Partum Hemorrhage (PPH). Uterine atony is one of the leading causes of PPH. Specialized interventions (uterine massage, starting Pitocin drip, etc.) may be needed to control PPH. Therefore, a dedicated nurse with postpartum unit-specific experience should be assigned to this client.

Choice D is incorrect. Soon after delivery, the uterine fundus (upper portion of the uterus), is midline and at 1 to 2 hours post-partum, it is palpable halfway between the symphysis pubis and the umbilicus. About 12 hours post-partum, the fundus is at the level of the umbilicus. In this scenario, at 5 hours postpartum, the client’s fundus is not yet at the midline. This means the fundus is displaced, and the most frequent cause of a displaced fundus is a full bladder. A full bladder may predispose to postpartum hemorrhage because it interferes with normal involution (contraction) of the uterus. The client should be asked to void. The medical nurse is usually not specialized in palpating the fundus, and therefore, this client should be assigned to a unit-specific experienced nurse.
NCSBN client need
Topic: Management of care; Sub-topic: Assignment, Delegation, and Supervision.
Reference
Pillitteri, A. Maternal and Child Health Nursing: Care of the Childbearing and Childbearing Family, 4th Edition; Lippincott, Williams & Wilkins, 2003

How well did you know this?
1
Not at all
2
3
4
5
Perfectly
10
Q

Which legislative initiative is the most closely related to information technology utilized in healthcare organizations?

A. The Health Insurance Portability and Accountability Act (HIPAA).

B. The state scopes of practice for the Informatics Nurse.

C. The Confidentiality and Information Security rule.

D. The Joint Commission for the Accreditation of Healthcare Organization’s ( JCAHO) standards.

A

Explanation

The correct answer is C.The Confidentiality and Information Security rule under the Health Insurance Portability and Accountability Act (HIPAA) is the legislative initiative that is the most closely and specifically related to information technology that is utilized in healthcare organizations. This rule specifically addresses the need for data security and protection.

Choice A is incorrect. The Health Insurance Portability and Accountability Act (HIPAA) is not the legislative initiative that is the most closely and specifically related to information technology that is utilized in healthcare organizations. There is another legislative initiative that is more specific to information technology than this.

Choice B is incorrect. The state scopes of practice for the Informatics Nurse are not legislative initiatives that are the most caring and specifically related to information technology utilized in healthcare organizations. Job descriptions for the Informatics Nurse may entail specifics about information technology and its security, but job descriptions are not legislated.

Choice D is incorrect. The Joint Commission for the Accreditation of Healthcare Organization’s ( JCAHO) standards address information technology and security; however, these are regulatory standards and not legislative initiatives.

Reference: Berman, Audrey, Snyder, Shirlee, and Geralyn Frandsen. Kozier and Erb’s Fundamentals of Nursing: Concepts, Process, and Practice.

How well did you know this?
1
Not at all
2
3
4
5
Perfectly
11
Q

Which of the following is the definition of death established in the Uniform Determination of Death Act of 1981?

A. Either irreversible cessation of circulatory and respiratory functions OR irreversible cessation of all functions of the entire brain including the brain-stem.

B. Both irreversible cessation of circulatory and respiratory functions AND irreversible cessation of all functions of the entire brain including the brain-stem.

C. Irreversible cessation of circulatory and respiratory functions only.

D. Irreversible cessation of all functions of the entire brain including the brain-stem only.

A

Explanation

Choice A is correct.

Important Fact:

The Uniform Determination of Death Act of 1981 defines death as either irreversible cessation of circulatory and respiratory functions OR the irreversible cessation of all functions of the entire brain, including the brainstem.

Choice B is incorrect because the Uniform Determination of Death Act of 1981 does not require both the cessation of circulation and respiratory functions AND irreversible end of all functions of the entire brain, including the brain stem.

Choices C and D are incorrect. Although C or D could constitute a death call, the Uniform Determination of Death Act of 1981 states that death is defined as EITHER option “C” or option “D” above. Hence, C and D are incorrect because these options use the term “Only.”

NCSBN Client Need
Topic: Physiological Integrity; Subtopic: Physiological Adaptation
Reference:
Fundamentals of Nursing/ Theories, Concepts, and Applications (Wilkinson/Treas/Barnett/Smith); Chapter17: Loss, Grief, and Dying; Lesson: Death and Dying

How well did you know this?
1
Not at all
2
3
4
5
Perfectly
12
Q

As you are taking the “staff only” elevator, you see a nurse who is now taking care of a client, Mr. B, who you cared for the week before. You ask the nurse how Mr. B is doing and the nurse tells you how significantly his condition has deteriorated over the last week. You have:

A. Violated the confidentiality of client information.

B. Asked an inappropriate question in the elevator.

C. Shown compassion for Mr. B.

D. Shown your caring about Mr. B.

A

Explanation

Correct Answer is B. You have asked an inappropriate question in the elevator. You have primarily set the other nurse up for a violation of the need for confidential client information because client information can only be shared, orally, and in writing, with others who are providing direct or indirect care to the client, and they have a need to know this information. As based on the information in this question, you are no longer taking care of Mr. B. Therefore; you should never have asked these questions.

The nurse who gave you the information violated Mr. B’s right to confidentiality. Although you asked this question because you are a compassionate and caring nurse, it was not an appropriate question.

Choice A is incorrect. You have not violated the confidentiality of client information because you did not share any client information with anyone.

Choices C and D are incorrect. Although you may have asked this question because you are a compassionate and caring nurse, this is not an appropriate question since you are no longer involved in the client’s care.

References: Kozier and Erb’s Fundamentals of Nursing: Concepts, Process and Practice and Sommer, Johnson, Roberts, Redding, Churchill et al.

How well did you know this?
1
Not at all
2
3
4
5
Perfectly
13
Q

The nurse is preparing medications for the shift. Which of the following clients should be prioritized for immediate medication administration?

A. Digoxin to a client with an apical pulse of 50.

B. Furosemide to a client with a serum potassium level of 3.0 mEq/L.

C. Magnesium sulfate to a client with Torsades de pointes

D. Verapamil to a client with blood pressure of 100/60 mm Hg.

A

Explanation

Choice C is correct.
Torsades de pointes, a form of ventricular tachycardia, is a life-threatening condition. The nurse should immediately administer the medication to the client to prevent the disease from progressing into ventricular fibrillation.

Choice B is incorrect. Furosemide is a loop diuretic used to treat congestive heart failure and edema. The drug predisposes the client to hypokalemia. In this case, the client already has a low serum potassium level. Therefore, the nurse needs to notify and question the prescribing physician whether he/she should still proceed with administering the medication.

Choice A is incorrect. When the nurse is administering digoxin, she should check the patient’s apical pulse and withhold the dose if the pulse falls below 60 beats per minute.

Choice D is incorrect. The blood pressure of the client is at 100/60 mm Hg. Verapamil is a calcium channel blocker and is often used to treat high blood pressure and angina. It can be administered as ordered. Typically, physicians order blood pressure medications to be held at a systolic blood pressure of 90 mm Hg or below. However, the nurse should prioritize administering magnesium to the client with Torsades de pointes.
Reference
Black, JM, Hawkes, JH; Medical-Surgical Nursing: Clinical Care for Positive Outcomes 8th edition, Nebraska: Elsevier.

How well did you know this?
1
Not at all
2
3
4
5
Perfectly
14
Q

The nurse is triaging a group of clients in the emergency department (ED). Which client should the nurse triage as emergent? Select only one option

A. 36-year-old complaining of pleuritic chest pain with a productive cough.

B. 81-year-old complaining of nausea and vomiting for two days.

C. 50-year-old presenting with malaise and an isolated area of reddened vesicles.

D. 68-year-old presenting with ataxia and dysarthria.

A

Explanation

Choice D is correct. A client presenting with ataxia and dysarthria is likely to have a cerebrovascular accident (CVA). This client is presenting with life-threatening symptoms and should be triaged as emergent. If the client is confirmed to have an ischemic stroke, the physician is likely to order reperfusion therapy (tPA)within a 3 to 4.5-hour window. Therefore, this presentation is an emergency.

Choice A is incorrect. The client with pleuritic chest pain and productive cough is likely presenting with pneumonia. There is no mention of unstable vitals. Such a client would be triaged as urgent. Per guidelines, such clients should receive antibiotics within 2 hours of presentation in the ED. This client should be attended after the client in Option D.

Choice B is incorrect. The elderly client with nausea and vomiting would be triaged as urgent. Such clients are prone to dehydration and renal failure. There is no mention of unstable vitals. The client should be attended after the client in option D.

Choice C is incorrect. The client with malaise and an isolated area of reddened vesicles is likely to have Shingles or Herpes Zoster. This client is triaged as non-urgent and should be attended to after other clients in options D, B, and A are attended.

How well did you know this?
1
Not at all
2
3
4
5
Perfectly
15
Q

During a busy shift, the nurse appropriately delegates tasks to the unlicensed assistive personnel (UAP) working with her. After charging, which of the following is the nurse’s primary responsibility?

A. Document the completion of the task

B. Make a list of tasks not yet completed to pass on to the next shift

C. Observe the UAP for the duration of the task

D. Follow-up with the UAP to ensure completion of the task, evaluating the outcome.

A

Explanation

D is correct. The nurse should follow-up with the UAP to ensure completion of the task, evaluating the outcome. The ultimate responsibility for any job will always remain with the person who delegated it. Therefore, after delegating a task, the nurse’s primary responsibility will be to follow up with the UAP.

A is incorrect. The nurse’s primary responsibility after delegating a task will be to follow up with the UAP. The nurse cannot document the completion of the job until the follow-up has been performed.

B is incorrect. It is unnecessary to make a list of tasks not yet completed to pass on to the next shift. The nurse’s primary responsibility after delegating a job will be to follow up with the UAP.

C is incorrect. If delegating correctly, the nurse must delegate a task within the scope of practice of the UAP and therefore does not need to observe the UAP for the duration of the job. The nurse’s primary responsibility after delegating a task will be to follow up with the UAP.

NCSBN Client Need:

Topic: Effective, safe care environment; Subtopic: Infection control and safety

Subject: Fundamentals; Lesson: Prioritization, delegation, and leadership

Reference: Cooper, K., & Gosnell, K. (2019). Study Guide for Foundations and Adult Health Nursing-E-Book. Elsevier Health Sciences.

How well did you know this?
1
Not at all
2
3
4
5
Perfectly
16
Q

ou are a nurse working in a medical unit with a trained aide. You have admitted a new patient and have received the following orders.You know that the correct prioritization for performing these tasks is: ( Please arrange in correct sequential order)
Vital Signs every 4 hours.
Insulin 2 units Humulin Subcutaneous now.
CBC, electrolytes, urinalysis, and 2 sets of blood cultures.
Amoxicillin 250 mg by mouth first dose now and then every 6 hours.

A

Explanation

Correct Sequence is in the following order:-

Insulin – 2 units Humulin Subcutaneous NOW.
CBC, Electrolytes, urinalysis, and 2 sets of blood cultures
Amoxicillin 250 mg by mouth first dose now, and then every 6 hours
Vital Signs every 4 hours

While prioritizing the orders from physician, the nurse should look for the orders that specify urgency – such as “STAT” or “as soon as possible” or “now.”

A “now” prescription for insulin should be done as soon as possible after the patient arrives on the floor. The nurse should understand that insulin lowers the patient’s blood sugar and can help to prevent sequelae associated with high blood sugar. Since the patient is being initiated on antibiotics, it appears there is a suspicion of infection. In patients with suspected infection, glycemic control is helpful in achieving good outcomes.

Collecting the labs is the second task that should be completed since blood cultures have been ordered. Blood cultures must always be collected BEFORE the administration of an antibiotic so that the antibiotic does not interfere with the results. Obtaining cultures after antibiotics may give false negative results.

As soon as the blood cultures are drawn, the nurse should administer the amoxicillin since it is ordered “now”, and every 6 hours. In almost any infection including sepsis, guidelines allow 1 to 2 hours window from the time of patient arrival before which antibiotics can be administered. Blood cultures must be obtained before antibiotics.

Finally, vital signs are the lowest priority for the nurse since this is a task that can be delegated to the aide following an initial assessment. It can be executed after the above orders are completed.
NCSBN Client Need
Topic: Management of Care;Sub-Topic: Establishing Priorities
Subject: Adult Health;Lesson: Prioritization

How well did you know this?
1
Not at all
2
3
4
5
Perfectly
17
Q

The nurse checks the history of several pre-operative patients before their scheduled surgeries. Which of the following patients should the nurse be most concerned with and alert the doctor about the elevated risk of surgical complications? Select all that apply.

A. The epileptic patient who took carbamazepine early in the morning

B. The diabetic patient with a blood glucose of 250 mg/dl.

C. The patient with anemia and a hemoglobin level of 6.5mg/dl.

D. The patient who suffers from insomnia.

E. The patient who reports a history of trouble being anesthetized

A

Explanation

Choices B and C are correct. The most concerning patients are diabetic patients with uncontrolled hyperglycemia and those with severe anemia. Both conditions significantly increase the patients’ risk of developing surgical complications and should be managed before surgery.

Hyperglycemia is an independent marker of poor surgical outcomes in both diabetic and non-diabetic patients. The random glucose test in an average adult normally ranges between 80mg/dl to 140mg/dl. A random blood sugar greater than 180 to 250 mg/dl is considered severe hyperglycemia. Uncontrolled hyperglycemia increases the risk of infections, delays surgical wound healing, prolongs hospital stay, and increases postoperative mortality. The physician must be notified, and the blood sugars must be optimized before surgical intervention. A desirable goal in most perioperative patients is to maintain blood glucose in the range of 140 to 180 mg/dl. In cases of severe hyperglycemia (greater than 250 mg/dl), surgery should be postponed by a few hours to obtain good glycemic control.

Severe anemia must be corrected before the patient undergoes surgery. Surgical blood loss may further worsen the pre-existing anemia. Severe anemia increases the risk of postoperative mortality. In patients with underlying cardiovascular disease, the risk of post-operative death significantly increases when preoperative hemoglobin is 10 g/dL or less. The physician should be alerted, so the cause of anemia is investigated, and transfusions are given as needed. In patients with no symptoms from anemia and no history of ischemic heart disease, hemoglobin above 7gm% is considered reasonable to undergo surgery. Those patients who are symptomatic from their anemia should be transfused as needed.

Choice A is incorrect. While many medications should be held before surgery, anti-convulsant such as carbamazepine should not be withheld. Post-operative electrolyte imbalances such as hypomagnesemia can increase the seizure potential in a patient with epilepsy. Anti-convulsant must never be withheld peri-operatively. If the patient ends up having a seizure intra-operatively or post-operatively, surgical outcomes may worsen.

Choice D is incorrect. Insomnia is not an absolute contraindication to performing surgery. Good sleep may help promote wound healing, and therefore, measures to improve sleep can be deployed after surgery.

Choice E is incorrect. While the nurse should inform the doctor regarding the prior history of the patient’s difficulty being anesthetized, this by itself does not increase the risk of surgical complications.

NCSBN client need
Topic: Reduction of Risk Potential: Potential for Complications for Surgical Procedures and Health Alterations.
Reference:
Lewis S, Dirksen S, Heitkemper M, Bucher L, Harding M. Medical-Surgical Nursing.

How well did you know this?
1
Not at all
2
3
4
5
Perfectly
18
Q

A 23-year-old college student seeks medical help at the infirmary complaining of severe fatigue. She reports exertional dyspnea, and her skin appears pale. Aplastic anemia is suspected. Laboratory values reflect anemia, and the client is advised for a bone marrow biopsy. The client refuses to sign the consent and states, “Come on, just get the doctor to give me a transfusion and let me go. Spring break begins this weekend, and I’m leaving for Florida.” The nurse’s most significant concern at this time would be:

A. The possibility that the client may contract infection from being exposed to large crowds at spring break

B. The client does not understand the full impact of her condition

C. The client may need transfusion before leaving for spring break

D. The causative agent needs to be identified and the treatment should be started

A

Explanation

Choice B is correct. The possibility of an infection is a concern but not the most pertinent issue at this point. The most significant concern at this point is the fact that the client does not fully grasp the gravity of her condition. She must be educated and be allowed to verbalize her feelings about her situation.

A transfusion is only a temporary measure because the causative agent has not been identified. For treatment to commence, a bone marrow biopsy needs to be done first, but before that, the client’s feelings regarding her condition need to be addressed for care to continue. The correct answer is option B. Options A, C, and D are incorrect.

Reference:

Ignatavicius DD, Workman LM. Medical-Surgical Nursing: Patient-Centered Collaborative Care, 7th ed. St. Louis, MO: Elsevier

How well did you know this?
1
Not at all
2
3
4
5
Perfectly
19
Q

The nurse and the Licensed Practical Nurse (LPN) are assigned to a busy medical unit. Which of the following tasks would be appropriate for an LPN to take? Select all that apply.

A. Reinforcing newborn care education to a 24-year-old first-time mother.

B. Adjustment of a 68-year-old stable patient’s cervical traction as ordered by the provider.

C. Obtaining a fecal occult blood sample from a 16-year-old patient with ulcerative colitis.

D. An assessment of a 36-year-old man newly admitted for chest pain.

A

Explanation

Choices A, B, and C are correct. Initial teaching does not fall within the scope of practice of an LPN. A registered nurse always performs initial instruction. However, LPNs can “reinforce” education (Choice A) to a client. Generally, the tasks that require “critical thinking” should not be delegated to an LPN. Tasks such as obtaining stool samples for occult blood (Choice B) and following health care provider’s orders to adjust cervical traction (Choice C) are all within the scope of practice of an LPN and do not require critical thinking process. LPNs can also apply and remove the cervical collar on stable spinal patients.

If the cervical traction is being applied for neck fractures, RNs or LPNs should not remove or add the traction weight since such patients have spinal instability. For an unstable client in traction, an RN should assess the neurovascular status, and document as ordered. Assessment or caring for unstable clients is not within the scope of LPN practice. However, cervical traction may also be applied for other reasons such as osteoarthritis, etc. In a client with stable clinical status and predictable outcomes, adjusting cervical traction as ordered by the provider falls within the scope of LPN practice. Choice B does not mention any unstable findings that fall outside the scope of an LPN.

For any question regarding delegation to LPNs, please make sure you determine the complexity and predictability of the client.

A Registered Nurse (RN) is responsible for determining the level of complexity and predictability of a client’s presentation. The RN documents this in an established plan of care. The LPNs accountability for the outcomes of care and independence of practice depends mostly on the predictability and complexity of the client presentation. Please note that the scope of practice is based on decisions around a task, not the job itself.

Predictability involves assessment of how effectively a health condition is managed, the changes likely to occur, and whether the type and timing of change can be predicted (College of Nurses of Ontario,1997, p.6). Complex or unstable situations are those where the patient’s status is fluctuating with unexpected responses resulting in an elaborate plan of care.

In cases where there is a high degree of complexity and a low degree of predictability, RNs are solely accountable for outcomes of care (“unstable” situations). In these cases, custody is determined by frequent assessments. The LPN practice in these cases is DIRECTED by the RN in that decisions of care are made by the RN only. Interventions change often, and patients’ responses to intrusions may be unexpected or high risk.
In acute cases where there are equal degrees of complexity and predictability, RNs and LPNs share accountability for the outcomes of care. LPN practice is COLLABORATIVE with the RN in that decisions of responsibility are made by the RN and LPN together.
In stable situations where there is a low degree of complexity and a high degree of predictability, the plan of care can be readily established. It can be managed with interventions that have predictable outcomes. Here, LPNs are solely accountable for the results of care. LPN practice is INDEPENDENT of the RN. The LPN is responsible for determining that the skills are appropriate for the patient.

Planning is not within the scope of an LPN. LPNs cannot formulate a care plan but may collaborate with an RN’s care plan.

In short, one may remember a popular mnemonic “DO NOT DELEGATE WHAT YOU EAT (LPNs cannot Evaluate, Assess, Teach).”

Choice D is incorrect. LPNs can perform focused assessments. However, initial and comprehensive assessments should always be performed by a registered nurse or an attending physician. The client with chest pain may involve a high degree of complexity and a low degree of predictability. Such assessment and planning require the critical thinking process.

Here is a 5-minute refresher video on the LPN Scope of Practice:

https://www.youtube.com/watch?v=EkYe7rSsJkk

How well did you know this?
1
Not at all
2
3
4
5
Perfectly
20
Q

The nurse is caring for a 2-year-old client who is intubated and mechanically ventilated. Two hours into the shift, the hospital receives a tornado warning. What is the priority action the nurse should take?

A. Clock out, her shift is over and she is not responsible.

B. Remove the child from the ventilator and carry her to a tornado shelter.

C. Move the patient as close to the interior of the room as possible

D. Close all of the doors

A

Explanation

Choice C is correct.
The priority action for the nurse is always to “best protect her patient.” During a tornado warning, the appropriate nursing action is to move patients away from windows and as close to the room’s interior as they can safely be. This action best protects them in the event of a tornado.

Choice A is incorrect. It is not appropriate to clock out because her shift is over. The nurse is always responsible for her patients’ safety, and clocking out does not best protect her patient.

Choice B is incorrect. It is inappropriate to remove the child from the ventilator because it could result in serious harm and even death if the child is dependent on mechanical ventilation.

Choice D is incorrect. Closing all of the doors will not protect the patient during a tornado. This is an appropriate action in some fire events depending on the fire’s location, but never for a tornado.
NCSBN Client Need:
Topic: Effective, safe care environment;Subtopic: Infection control and safety

Reference: Hockenberry, M., Wilson, D. & Rodgers, C. (2017). Wong’s Essentials of Pediatric Nursing (10th ed.) St. Louis, MO: Elsevier Limited.

How well did you know this?
1
Not at all
2
3
4
5
Perfectly
21
Q

A patient has completed a living will stating that he does not want intubation, mechanical ventilation or artificial nutrition/ hydration should he become unable to communicate his preferences related to medical care. However, the patient’s adult children have expressed their opposition to the patient’s wishes. Which are appropriate nursing actions? Select allthat apply.

A. Notify the patient’s physician, the nursing supervisor, and the risk manager.

B. Explain to the patient’s family that the living will cannot be changed at this point.

C. Encourage the family to discuss their feelings to try to resolve this issue.

D. Request a consult with the facility ethics committee if needed.

E. Advise the patient to just go along with the wishes of his adult children.

A

Explanation

Correct answers are A, C, and D. Should such a conflict be observed, the nurse should notify the patient’s physician, the nursing supervisor and the risk manager, and encourage the family to discuss the issue among themselves, and with the above individuals, to resolve the conflict. A consult with the ethics committee may also be indicated.

Choice B is incorrect. The patient may revoke or change an advance directive at any time, either orally or in writing.

Choice E is incorrect. By law, the patient has a right to autonomy and self-determination, including the right to choose and refuse treatment.

Blooms Taxonomy - Analyzing
References:
Potter, P., Perry, A., Stockert, P., Hall, A., Fundamentals of Nursing 8th Edition. Elsevier Mosby St Louis 2013.
Lippincott’s Nursing Procedures 5th Edition. Lippincott Williams & Wilkins. 2009

How well did you know this?
1
Not at all
2
3
4
5
Perfectly
22
Q

The nurse is providing patient care working in a unit that uses the total patient care model for delivering nursing care. The nurse recognizes which of the following as an aspect of this nursing care delivery model?

A. The RN assumes responsibility for a caseload of patients

B. The RN supervises team members providing direct patient care

C. The RN provides care for the same patients during their hospital stay

D. The RN is responsible for all aspects of care during a shift of care

A

Explanation

Correct Answer is D. Characteristics of the total patient care model include; the RN being responsible for all aspects of care during a shift of care, care can be delegated, and the RN works directly with the patient, family, and health care team members.

Choices A and C are incorrect.The RN having responsibility for a caseload of patients and providing care for the same patients during their hospital stay are characteristics related to the primary nursing model.

Choice B is incorrect. In team nursing, team members provide patient care under the supervision of the RN team leader.

Bloom’s Taxonomy: Analyzing.
Reference:
Potter, P., Perry, A., Stockert, P., Hall, A., Fundamentals of Nursing 8th Edition. Elsevier Mosby St Louis 2013.

How well did you know this?
1
Not at all
2
3
4
5
Perfectly
23
Q

The patient is presenting with a fever, nausea, and dysuria. Which action would the nurse take first?

A. Administer as needed antipyretic.

B. Call physician to obtain an antibiotic order for a suspected UTI

C. Collect midstream, clean-catch urine specimen

D. Collect STAT blood cultures

A

Explanation

C is correct. The nurse should recognize that this patient is presenting with symptoms of urinary tract infection (UTI) or pyelonephritis. The most appropriate first action would be to assess the patient and check the urine for infection.

A is incorrect. In a patient who is not in distress or severe pain, the nurse should not administer medication until the assessment is complete.

B is incorrect. The nurse should finish assessment prior to calling physicians since there is no data to support a medical emergency scenario to immediately notify physician without a complete assessment. Besides, urine specimen should be collected before administering an antibiotic in a suspected UTI.

D is incorrect. The patient’s symptoms are consistent with urinary tract infection and would not necessarily warrant blood cultures. A complicated UTI may evolve in to sepsis. If signs of sepsis are present, blood cultures would be appropriate.

NCSBN Client need:
Topic: Establishing priorities, system-specific assessment

Reference: (DiGiulio & Keogh, 2014, p. 368);Subject:Adult health;Lesson:Renal/urinary

How well did you know this?
1
Not at all
2
3
4
5
Perfectly
24
Q

The nurse on a medical floor receives a report on four patients. Which patient should the nurse see first?

A. A client with pulmonary embolism on anticoagulation, dyspnea, and pCO2 of 30mmHg

B. A client with atrial fibrillation on Warfarin, history of prior rectal bleeding and an INR of 6.0

C. A client Congestive Heart Failure and Brain Natriuretic Peptide of 640 pg/mL

D. A client with Acute pancreatitis and serum calcium of 8.9 mg/dL

A

Explanation

Choice B is correct. While answering prioritization questions, it is essential to determine which findings are unexpected and which pose an immediate risk of complications to the client. The target International Normalized Ratio (INR) for atrial fibrillation is 2.0-3.0. A supra-therapeutic INR of 6.0 is too high for this patient and puts the patient at high risk for bleeding. Additionally, given his prior history of gastrointestinal bleeding, he is more prone to recurrent bleeding in the setting of coagulopathy. The nurse should hold warfarin, assess the patient for signs of bleeding and notify the physician of abnormal results to determine if vitamin K should be administered to counter the effects of warfarin.

Choice A is incorrect. The client has an established diagnosis of Pulmonary Embolism (PE) and is on therapeutic anticoagulation. Dyspnea and elevated D-dimer are expected results in patients with known PE. D-dimer reflects thrombin and plasmin activity and is usually positive in hospitalized patients with thrombotic events. Low pO2 (Hypoxia) and low pCO2 (Respiratory alkalosis) are expected findings in patients with PE. Normal PCO2 is 35-45 mmHg, so 30 mmHg is small but not critical (<20 mmHg).

Choice C is incorrect. Brain Natriuretic Peptide is a marker for Congestive Heart Failure (CHF) because it correlates with left ventricular pressure. High Left ventricular pressures and high BNP levels are expected findings in patients with heart failure. A BNP more top than 100 pg/mL is abnormal. The client has an established diagnosis of CHF, and a report of BNP at 640 pg/mL does not require immediate action.

Choice D is incorrect. Acute pancreatitis can cause decreased calcium levels (hypocalcemia). Severe hypocalcemia may be seen in acute pancreatitis and can present with neurological as well as cardiovascular manifestations. However, since the normal range for serum calcium level is 8.6-10.2 mg/dL, this patient’s result of 8.9 mg/dL is within normal range and would not warrant any intervention.

NCSBN Client Need:
Subject: Leadership/management; Lesson: Prioritization

Reference: (Lewis, Dirksen, Heitkemper, Bucher, & Camera, 2011, p. 578-579)

How well did you know this?
1
Not at all
2
3
4
5
Perfectly
25
Q

The nurse is studying the role of referrals in coordinating the client’s care. The primary purpose of referrals is to:

A. Ensure that the continuum of care is a seamless transition.

B. Ensure the completeness and appropriateness of the client care.

C. Establish the registered nurse as the center of client care.

D. Establish the client or the group as the center of client care.

A

Explanation

Correct Answer is B.The primary purpose of referrals is to ensure the completeness and appropriateness of the client care. Although there are many clients’ needs that the nursing team can address, there are also client needs that can be met by others in the multidisciplinary healthcare team.

Choices A, C, and D are incorrect. Although the continuum of care must be a seamless transition, and referrals are an essential component of client movement along the continuum of care, it is not the primary purpose of references.

Lastly, the client is the center of the client care, not the registered nurse. Although clients participate in the referral process, establishing the client or the group as the center of client care is not the primary purpose of referrals.

Reference: Ellis, Janice Rider, and Celia Love Harley (2012). Nursing in Today’s World: Trends, Issues, and Management (10th Edition). Philadelphia, PA: Lippincott Williams and Wilkins.

How well did you know this?
1
Not at all
2
3
4
5
Perfectly
26
Q

The nurse is taking care of a client receiving a D5LR intravenous infusion. Suddenly, the client complains of chest pain and difficulty breathing. On exam, there is cyanosis and tachycardia. The nurse also notices an empty IV bottle. What is the initial intervention of the nurse?

A. Replace the empty IV bottle with a new one.

B. Replace the IV line and attach a new IV bottle.

C. Stop the IV infusion and turn the client on his left side with the head of the bed lowered.

D. Stop the IV infusion and notify the physician.

A

Explanation

Choice C is correct. The nurse should suspect “air embolism” because the patient is presenting with characteristic symptoms in a setting where the fluid infusion is complete and the IV drip set is still open. An empty IV “bottle” offers this clue. Manifestations of an air embolism include tachycardia, hypotension, chest pain, the difficulty of breathing, and cyanosis. Air embolism may cause blockage of small pulmonary vessels compromising the gas exchange, obstruction of ventricular pumping, and arrhythmias. In practice, replacement of IV infusion bottles with collapsible air bags has largely minimized the risk of air embolism during IV infusions.

The nurse’s initial action would be to turn off the infusion system, place the client on his left side with the head lowered (left Trendelenburg position), and notifying the physician. Left-sided Trendelenburg position will help the air bubble float in the right ventricle/ right atrium and prevents it from causing right ventricular outlet obstruction.

Choice A is incorrect. Replacing the IV bottle is not an appropriate intervention in this situation since the client is already presenting with signs and symptoms of air embolism. To prevent air embolism, the nurse should have replaced the IV bottle before it is empty. At this time, the nurse should turn off the infusion system and place the client in left-Trendelenburg position.

Choice B is incorrect. Replacing the IV line and hooking up a new IV bottle does not address the air embolism.

Choice D is incorrect. A nurse should never delay a lifesaving intervention which is within the scope of his/her practice. After stopping the infusion, the next immediate action for the nurse would be to place the client on his left side with the head lowered to trap the air in the right atrium. Following this, the nurse must notify the physician.
Reference
Ignatavicius DD, Workman LM. Medical-Surgical Nursing: Patient-Centered Collaborative Care, 7th ed. St. Louis, MO

How well did you know this?
1
Not at all
2
3
4
5
Perfectly
27
Q

Which of the following components should the nurse know to include in her handoff at the end of her shift?

A. List of p.r.n (as needed) medications administered

B. Normal assessment findings for the shift

C. A complete history of lab results and interventions since admission

D. All scheduled medications the client receives.

A

Explanation

Choice A is correct. Medications administered as needed should be included in the nursing handoff. Nursing handoff should review the client’s condition during the past shift accurately, but quickly. It is important to include important information about the client about what has occurred over the previous change, and if any “as needed” (p.r.n) medications were administered.

B is incorrect. Normal assessment findings for the shift are not a necessary component of the nursing handoff. Nursing handoff should review the client’s condition during the past shift accurately, but quickly. Reviewing all normal assessment findings would not only take too long but is not necessary information. Any changes in assessment findings, abnormal findings, and current problems should be included, but normal assessment findings are not required to cover.

C is incorrect. A complete history of lab results and interventions since admission is not a necessary component of the nursing handoff. Nursing handoff should review the client’s condition during the past shift accurately, but quickly. Going over a complete history of lab results and interventions since admission would not only take too long but would not be pertinent.

D is incorrect. All scheduled medications the client receives is not a necessary component of the nursing handoff. Nursing handoff should review the client’s condition during the past shift accurately, but quickly. As needed medications, changes in the client’s situation, interventions, and the client’s response to such interventions are part of the nursing handoff.

NCSBN Client Need:

Topic: Effective, safe care environment; Subtopic: Coordinated care

Subject: Fundamentals; Lesson: Prioritization, delegation, and leadership

Reference: Cooper, K., & Gosnell, K. (2019). Study Guide for Foundations and Adult Health Nursing-E-Book. Elsevier Health Sciences.

How well did you know this?
1
Not at all
2
3
4
5
Perfectly
28
Q
While working in an adult cardiac telemetry unit, you see each of the following rhythms on the monitor. Which of the following is your immediate priority?
A . 3RD-DEGREE BLOCK
B . SINUS BRADYCARDIA
C . SUPRAVENTRICULAR TACHYCARDIA
D . NORMAL SINUS RHYTHM
A

Explanation

Choice A is correct. This Electrocardiogram (ECG) shows a third-degree heart block, otherwise known as a complete heart block. This rhythm is fatal without intervention. If you notice this, you must notify the physician immediately so appropriate interventions can be implemented.

A complete heart block may lead to fatal symptomatic bradycardia with a heart rate of less than 40/min, hypotension, seizures, cerebral ischemia, or cardiac arrest, and sudden cardiac death. It, therefore, is considered a fatal rhythm, and hence, this patient is a top priority.

To understand a complete heart block, you must first understand a normal EKG. The image shown below indicates a normal EKG where there is a clear relationship between a “P” wave and “QRS” complex. P-R interval is regular. “P” wave indicates SA (sinoatrial node) node firing, whereas QRS indicates Ventricular contraction. PR interval is the time interval from Sinus node firing to the time it reaches ventricle and results in shrinkage. “T” wave indicates ventricular repolarization, and it marks the beginning of ventricular relaxation.

In a 3rd degree or complete heart block, there is no atrioventricular conduction, so no impulses from the supraventricular nodes (sinus impulses) are conducted to the ventricles whatsoever. This results in erratic heart rates where the sinus node and the atrioventricular nodes are beating independently. This leads to a junctional rhythm where there is no correlation between P-waves and QRS complexes. The atrial rhythm will be regular (P to P interval regular). Ventricular rhythm is steady (R-to-R range is consistent). However, the P-R interval will be variable. These are the typical characteristics of a 3rd degree AV block.

Choice B is incorrect. This ECG shows sinus bradycardia. There are a regular P-R interval and the proper relationship between P waves and QRS complexes. But the heart rate is lower. The price of impulses arising from the sinoatrial (SA) node is more economical than expected. The average adult heart rate, resulting from the SA node, is usually 60 to 100 beats per minute. Sinus bradycardia is defined as a sinus rhythm with a rate below 60 beats per minute. While sinus bradycardia can sometimes be concerning, it is not a fatal rhythm that requires immediate attention in the absence of symptoms. This may even be an average heart rate for the patient, depending on their baseline. Athletes who are very well conditioned tent to have low baseline heart rates, such as a heart rate in the 40s. Therefore, it is so important to know what is normal for your patient.

Choice C is incorrect. This ECG shows supraventricular tachycardia or SVT. While this is a concerning rhythm that will need attention, it is not your priority given that you have a patient in third-degree heart block that could suffer sudden cardiac death. SVT is often tolerated by patients for long periods and can be challenging to break. The typical treatment is adenosine or cardioversion to convert the patient back into normal sinus rhythm.

Choice D is incorrect. This ECG shows normal sinus rhythm or NSR. This is the baseline rhythm that we expect to see in all patients, so this patient would not be your priority. You would need to see all the other patients on your above list first, as they all have irregular rhythms.
NCSBN Client Need:
Topic: Physiological Integrity; Subtopic: Physiological adaptation

Reference: DeWit, S. C., Stromberg, H., & Dallred, C. (2016). Medical-surgical nursing: Concepts & practice. Elsevier Health Sciences.

How well did you know this?
1
Not at all
2
3
4
5
Perfectly
29
Q

You are taking care of an 80-year-old patient who is post-op day one from abdominal surgery. Upon assessment, you notice bowel protruding through her incision and quickly determine that evisceration has occurred. Place the following actions in order of priority:
Prepare the patient for immediate surgery
Take vital signs and monitor for and signs of shock
Call for help and stay with the patient
Document the incident.
Cover the wound with a sterile normal saline dressing

A

Correct Answer is:
Call for help and stay with the patient
Cover the wound with a sterile normal saline dressing
Take vital signs and monitor for and signs of shock
Prepare the patient for immediate surgery
Document the incident.

Explanation

The priority of nursing action is to call for help but stay with the patient. The nurse should tell the person who responds to notify the surgeon immediately. This is a surgical emergency, and the surgeon must be notified STAT.

After help has been called, the nurse needs to cover the wound with a sterile 0.9% sodium chloride dressing. This helps prevent infection and keep the protruding organ moist and hydrated before surgery. The nurse should instruct the patient not to strain or cough, and keep the client in low Fowler’s position ( no more than 20 degrees bed elevation) with his/her knees flexed. This position relaxes abdominal muscles and reduces abdominal muscle tension.

After these two actions, the next nursing action is to check the patient’s vital signs and monitor for shock while waiting for the health care providers.If signs of shock such as tachycardia and hypotension are noted, there is a medical emergency, and the health care provider/ rapid response needs to be called to the bedside immediately.

After taking vital signs, the nurse should begin preparing the patient for immediate surgery. Lastly, after the patient has been taken to surgery, the nurse needs to document the incident.
NCSBN Client Need
Topic: Physiological Adaptation Subtopic: Medical Emergencies
Reference:
Ignatavicius D, Workman ML: Medical-surgical nursing: Patient-centered collaborative care, ed 7, Philadelphia.

How well did you know this?
1
Not at all
2
3
4
5
Perfectly
30
Q

The nurse receives a call from her mother who tells her that her father is having sudden and severe chest pain but is refusing to go to the hospital. What should be the nurse’s initial action?

A. Tell her mother to call 911.

B. Ask her mother to let her father chew an aspirin.

C. Ask what her father ate recently.

D. Ask her mother if she can talk to her father.

A

Explanation

Choice B is correct. An acute myocardial infarction (MI) should always be suspected while considering a differential diagnosis of new-onset chest pain in an elderly man. Often, the pathophysiology of acute myocardial infarction or unstable angina involves thrombus (clot) formation and propagation. The patient should be given an aspirin to chew on first to decrease platelet aggregation and prevent the thrombus from getting any bigger. The patient is obviously in distress and intervention should be initiated. The benefits of early aspirin administration outweigh the risks. Several clinical trials have strongly suggested that the early administration of aspirin can significantly reduce the size of the myocardial infarction and improve survival. Studies have shown that aspirin administration within one hour of emergency room presentation by a patient with an acute MI or unstable angina is associated with a substantial reduction in 30-day mortality when compared to those given aspirin an hour after the presentation. Therefore, the nurse should certainly advise her mother to give her father an aspirin. Also, the patient should be instructed to chew aspirin rather than swallowing it at once. Chewed aspirin works faster compared to swallowing it at once.

Generally, if a patient experiences a new onset chest pain, the recommended sequence of action is to call 911 or local emergency first and then take an aspirin. Often, it is the 911 operator who first evaluates the patient’s symptoms and advises the patient to take an aspirin. Once dispatched, the paramedics can give oxygen and medications and move the patient fast to the ER. If the patient is indeed having a heart attack, early intervention with thrombolytics or angioplasty can limit the damage. Therefore, calling 911 is crucial. In this case, the patient’s wife has already called her daughter who is a nurse. The wife also mentioned that the patient is unwilling to go to the hospital. Being a nurse, it would be an appropriate and responsible action for the daughter to advise her mother to give the aspirin right away. Following that, the daughter can talk to her father to alleviate his anxiety, convince him to call 911, and go to the hospital.

Choice A is incorrect. Had the mother not already reached out to her daughter who is a nurse, the recommended course of action would be to call 911 and then give her husband an aspirin. However, the mother has reached out to a nurse. In the role of a nurse, it is appropriate for the daughter to advise her mother to give her father aspirin right away and then call 911.

Choice C is incorrect. Obtaining more history will certainly help to narrow down the differential diagnoses of new-onset chest pain. Asking what her father ate could give the nurse additional information about the potential cause of the chest pain; if it is related to a gastrointestinal or cardiac etiology. However, the patient is in distress and the first intervention should be to give an aspirin. The benefits of early aspirin administration outweigh the risks.

Choice D is incorrect. The nurse can talk to her father to alleviate his anxiety and get more history while her mother is getting the aspirin. The priority action should be to give aspirin to her father.

Reference: Black, JM, Hawkes, JH; Medical-Surgical Nursing: Clinical Care for Positive Outcomes, Nebraska: Elsevier.

How well did you know this?
1
Not at all
2
3
4
5
Perfectly
31
Q

Case management, as a form of patient care delivery and documentation, is most closely aligned with:

A. A problem-oriented documentation system.

B. A critical pathway documentation system.

C. A source-oriented documentation system.

D. A variance-oriented documentation system.

A

Explanation

Correct Answer is B. Case management, as a form of patient care delivery and documentation, is most closely aligned with a critical pathway documentation system. Critical pathways are time-oriented multidisciplinary plans of care that are established and approved by the interdisciplinary team.

Choice A is incorrect. The problem-oriented documentation system is not used with case management as a form of patient care delivery and documentation. The problem-oriented documentation system can be used with other types of patient care delivery and literature, but not case management.

Choice C is incorrect. The source-oriented documentation system is not used with case management as a form of patient care delivery and documentation.

Choice D is incorrect. There is no variance-oriented documentation system even though differences are determined and documented on the critical pathway, which is used with case management as a form of patient care delivery and documentation.

Reference: Berman, Audrey, Snyder, Shirlee, and Geralyn Frandsen. Kozier and Erb’s Fundamentals of Nursing: Concepts, Process, and Practice.

32
Q

The characteristics of the crisis include which of the following? Select all that apply.

A. A prolonged period of time occurs before the actual anticipated crisis.

B. Crises result from anticipated life threatening events.

C. A crisis results from a rapid and unanticipated life threatening event.

D. Crises result from actual and perceived threats to the person.

E. Crises can be quite brief and self-limiting in term of their nature.

A

Explanation

Correct Answers are C, D, and E.
A crisis results from a rapid and unanticipated life-threatening event; crises can be precipitated in response to both actual and perceived threats to the person, and emergencies can be quite brief and self-limiting in terms of their nature.

Crises are typically sudden and without the needed time to be able to cope with it, and they are most often unanticipated. They can occur as the result of an actual or perceived life-threatening event.

Choice A is incorrect. Crises are typically sudden and without the needed time to be able to cope with it.

Choice B is incorrect. Crises are most often unanticipated, and they can occur as the result of an actual or perceived life-threatening event.

References: Berman, Audrey, Snyder, Shirlee, and Geralyn Frandsen. (2016).

33
Q

When making patient care assignments, the nurse delegates care activities to nursing assistive personnel [NAP]. What factors must the nurse consider? Select all that apply.

A. Patient gender and ethnicity

B. Complexity of the tasks

C. Knowledge and skills of the NAP

D. Scope of practice for the NAP

A

Explanation

Correct Answers are B, C, and D. When delegating patient care activities to nurse assistive personnel; the RN must be aware of patient needs, the complexity of the tasks to be assigned, the knowledge and skills of the individual NAP, and which jobs are appropriate to delegate according to the scope of practice for NAPs.

Choice A is incorrect. Patient gender and ethnicity are not primary concerns.

Bloom’s Taxonomy: Applying
Reference:
Potter, P., Perry, A., Stockert, P., Hall, A., Fundamentals of Nursing 8th Edition. Elsevier Mosby St Louis 2013.

34
Q

A family unit you are caring for is experiencing a situational crisis that has led to dysfunctional communication within the family. You have recommended that the entire nuclear family and members of the extended family who live in the family’s home begin family therapy. The grandparents tell you that their grandson who is addicted to prescription painkillers is the cause of the problem. Because he is not their son, they feel that they do not have to participate in this group therapy. How should you respond to these grandparents?

A. “You should try to come to a few sessions at least because they may be very informative to you.”

B. “You are probably correct. This really is not your problem.”

C. “Despite the fact that it is your grandson’s drug addiction, situations such as this affect all members of the family including grandparents who live in the home.”

D. “You should attend because the doctor has ordered family therapy for you as extended family members”.

A

Explanation

Answer & Rationale:

The correct answer is C. You should respond to the grandparents’ statement with “Despite the fact that it is your grandson’s drug addiction, situations such as this affect all members of the family including grandparents who live in the home”. (After this statement, you should also educate the grandparents about the fact that group and family therapy is often indicated when the family unit is affected with stressors and dysfunction because family members may not fully understand the need for the entire family unit to participate when only one member of the family is adversely affected with a stressor and poor coping and that all family members are affected when only one member of the family unit is adversely affected.)
A is incorrect. You would NOT state “You should try to come to a few sessions at least because they may be very informative to you” because these sessions are therapeutic and not educational.
B is incorrect. You would not state “You are probably correct. This really is not your problem” because this statement is not true.
D is incorrect. You should also not state “You should attend because the doctor has ordered family therapy for you as extended family members” because this is not the real reason why attending these sessions is needed.

Resource

NCSBN Client Need

Topic: Psychosocial Integrity

Chapter 26: Communication

Lesson: Family Communication Patterns

Reference: Kozier and Erb’s Fundamentals of Nursing

35
Q

A client is currently experiencing bradycardia, low blood pressure, and dizziness. Which of the following does the nurse expect to be ordered?

A. Defibrillation

B. Digoxin

C. Monitor the client closely

D. Prepare patient for transcutaneous pacing

A

Explanation

Choice D is correct. The normal heart rate in an average adult is between 60 to 100 beats per minute. A heart rate less than 60 beats per minute is referred to as bradycardia. Bradycardia can be symptomatic or asymptomatic. Some healthy adults and athletes may have a heart rate between 40 and 60 beats per minute and do not experience any symptoms. When symptomatic, bradycardia can lead to shortness of breath, dizziness, and low blood pressure ( hypotension, shock). A patient experiencing symptomatic bradycardia will likely need transcutaneous pacing. In addition, an EKG must be performed to confirm the rhythm. The etiology of bradycardia may vary and include reversible ( medications) and irreversible causes ( heart blocks). Therefore, one should explore causes, but the priority intervention in a patient experiencing symptoms from bradycardia is to restore the heart rate quickly with transcutaneous pacing and maintain circulation.

Choice A is incorrect. Defibrillation is recommended when the patient is experiencing pulseless ventricular tachycardia or ventricular fibrillation.

Choice B is incorrect. Digoxin is a cardiac glycoside that has negative chronotropic action on the sinus node. Therefore, digoxin decreases the heart rate and would be dangerous in this patient with symptomatic bradycardia.

Choice C is incorrect. While this patient should be monitored closely, priority action ( transcutaneous pacing) should quickly restore the heart rate.

NCSBN client need | Topic: Physiological Integrity, Reduction of Risk Potential

36
Q

The nurse is assessing a client who has just returned from an exploratory laparotomy for peritonitis. The nurse checks preoperative vital signs (VS) at 10:00 AM to compare them with the current VS at 11:45 AM. Click the Exhibit button above for additional information.

Which action should the nurse take first?

A. Apply supplemental oxygen at the prescribed 2 liters per minute (L/min

B. Increase the intravenous fluid (IVF) rate, as prescribed.

C. Administer intravenous (IV) metronidazole, as prescribed

D. Collect the prescribed blood cultures.

A

Explanation

Correct Answer is B. The client’s blood pressure has decreased compared to the 10:00 AM reading.

Further, an increase in the pulse was noted. Hypotension and tachycardia are consistent with hypovolemia in this post-operative patient. In the presence of fever, this could also represent sepsis progressing towards septic shock. The most appropriate priority action here is to increase the prescribed intravenous fluids to restore the client’s volume.

Choice A is incorrect. There is no indication that the client has respiratory distress or hypoxemia. Oxygen saturation is at 95% and adequate.

Choice C is incorrect. Antibiotics should be initiated eventually, but restoring the client’s volume status is the priority action.

Choice D is incorrect. Fe636ver. Tachycardia and hypotension may suggest sepsis. Blood cultures should be done after the priority action in Choice B is performed to restore volume status. Blood culture is for diagnostic purposes and will not correct the low blood pressure.

37
Q

The nurse has been assigned the responsibility for all aspects of providing patient care during a 12-hour shift. How would you classify this approach to nursing care?

A. Team Nursing

B. Case Management

C. Total Patient Care

D. Primary Nursing

A

Explanation

Correct Answer is C. In the total patient care model, the RN assumes responsibility for all aspects of care for a patient or group of patients during a shift of attention, although care can be delegated. The RN works directly with the patient, family, and health care team members.

Choice A is incorrect. With team nursing, the RN leads a team consisting of other RNs, LPNs, and NAPs, with team members providing direct patient care under the supervision of the team leader.

Choice B is incorrect. Case management is an approach that coordinates health care services, linking patient/family to the needed services. Rather than providing direct patient care, the case manager typically supervises the care provided by the health care team.

Choice D is incorrect. In primary nursing, the RN is assigned responsibility for a caseload of patients, developing the plan of care, and when working, providing care for the workload of patients throughout their hospital stay.

Bloom’s Taxonomy: Analyzing
Reference:
Potter, P., Perry, A., Stockert, P., Hall, A., Fundamentals of Nursing 8th Edition. Elsevier Mosby St Louis 2013.

38
Q

The nurse receives a report on four patients at the change of shift. Which patient should the nurse see first?

A. Patient with right femur fracture who complains of right leg pain.

B. Patient being treated for pneumonia with scheduled IV antibiotic due.

C. Patient with history of T6 spinal injury 6 months ago, now presents with headache.

D. Patient that is 1-day postoperative open cholesystectomy with green drainage.

A

Explanation

Choice C is correct. This patient may be developing autonomic dysreflexia, a medical emergency. One of the first signs/symptoms of autonomic dysreflexia is a severe, throbbing headache following spinal cord injury (most common in T6 and above). Patients usually develop autonomic dysreflexia one month to one year after their injury. However, it has also been described in the first days or weeks after the original trauma. Objectively, an episode is defined as an increase in systolic blood pressure of 25 mm Hg. Patients with this condition will develop dangerously high blood pressure that can result in severe, fatal diseases such as seizures, pulmonary edema, and myocardial infarction. Assessing this patient would be the nurse’s highest priority.

Choice A is incorrect. Right leg pain is expected in a patient with an acute right femur fracture. The nurse needs to address this patient’s pain, but expected outcomes would not be the highest priority.

Choice B is incorrect. Scheduled medications would not be a higher priority than the patient showing symptoms of a life-threatening complication.

Choice D is incorrect. Green drainage is expected in a patient with an open cholecystectomy due to the green color of bile in the common bile duct. The nurse should assess this patient’s drainage and progression of healing, but it would not be the highest priority.

NCSBN Client need
Topic: Establishing priorities, medical emergencies

Reference: (Lewis, Dirksen, Heitkemper, Bucher, & Camera, 2011, p. 1558)

39
Q

A patient recovering from myocardial infarction is presenting with Heart rate 110 beats per minute, Blood Pressure 86/58 mmHG, crackles, shortness of breath, dusky skin, and jugular vein distention. Which action should the nurse recognize as the highest priority?

A. Administer medications to increase stroke volume.

B. Provide analgesics.

C. Obtain STAT Electrocardiogram and troponins

D. Administer fluid replacement to increase blood pressure.

A

Explanation

Choice A is correct. Based on the assessment information, the nurse can determine the patient is experiencing cardiogenic shock secondary to myocardial infarction. Since cardiogenic trauma occurs as a result of the heart not pumping effectively, the highest priority is to increase cardiac output to ensure adequate tissue perfusion.

Cardiac Output = Stroke volume x Heart Rate.

Medications that improve stroke volume will improve cardiac output in cardiogenic shock. The following agents may be used in the pharmacological management of cardiogenic shock.

Inotropes: Positive Inotropes strengthen the heart contractility (increase stroke volume). Dobutamine has more beta-adrenergic action than alpha activity. It causes peripheral vasodilation while increasing contractility. But in higher doses, it may increase heart rate and exacerbate myocardial ischemia.
Vasopressors: In severe shock, Vasopressors (Dopamine, Norepinephrine) maintain blood pressure but decrease blood flow to organs. They increase afterload and reduce cardiac output. However, they may be needed initially to provide hemodynamic support. Dopamine increases myocardial contractility and maintains blood pressure. If Dopamine fails to support blood pressure, norepinephrine is added.
Vasodilators: Vasodilators (Nitroglycerin) decrease venous return (preload) to the heart and decrease peripheral resistance (afterload). Although vasodilators may drop blood pressure, they sustain cardiac output and help achieve hemodynamic stability when combined with vasopressor support in cardiogenic shock.
Supplemental oxygen may also be necessary to increase tissue oxygenation.

Choice B is incorrect. There is no assessment information in the question that points to chest pain. If a patient in cardiogenic shock is showing signs or complaining of pain, this action would be appropriate, but not the highest priority.

Choice C is incorrect. The patient recently experienced MI, so they should already be on a telemetry monitor. ECG will likely be abnormal, and troponins may still be elevated. This action may be appropriate but will not change the immediate treatment of shock, so it would not be the highest priority.

Choice D is incorrect. Fluid replacement is not the correct immediate action because the patient is showing signs of pulmonary edema (crackles, shortness of breath, jugular vein distention). Cardiac output needs to be improved before considering the additional fluid volume. This action might be appropriate if the patient was in hypovolemic shock, not cardiogenic.

NCSBN Client Need
Topic: Establishing priorities, illness management, medical emergencies, pathophysiology,

Reference: (Jones & Fix, 2015, p. 234-236), (Lewis, Dirksen, Heitkemper, Bucher, & Camera, 2011, p. 1733), (Huether & McCance, 2008, p. 659)

40
Q

A registered nurse has encountered an ethical dilemma regarding euthanasia in the medical unit earlier in the day. The nurse verbalizes to the manager that she is concerned about what she witnessed. Does the manager suggest which resource for the RN to utilize?

A. Rights for the Mentally Ill

B. Client’s Bill of Rights

C. Code of Ethics

D. Nurse Practice Acts (NPA)

A

Explanation

Choice C is correct. The Code of Ethics for nurses provides ethical guidelines regarding nursing practice.

Choice A is incorrect. The Rights for the Mentally Ill provides people with mental illness the civil liberties that are due to them.

Choice B is incorrect. The Patient’s Bill of Rights outlines the rights that are due to them when admitted and seeking health care.

Choice D is incorrect. Nurse practice acts describe the scope of nursing practice. It directs the philosophy and standards of nursing.
Reference
Potter, PA, Perry. AG, Stockert, PA, Hall, AM. Fundamentals of Nursing, 8th ed. St. Louis, MO: Elsevier Mosby;2013

41
Q

Which reimbursement principle are you employing when you appropriately transfer a client from an acute care healthcare facility to a subacute healthcare facility along the continuum of care?

A. The principle of retrospective reimbursement.

B. The principle of rehabilitation care.

C. The need to limit lengths of stay.

D. The need for medical necessity.

A

Explanation

Correct Answer is D. The reimbursement principle that you are employing when you appropriately transfer a client from an acute care healthcare facility to a subacute healthcare facility along the continuum of care is the need for medical necessity. Healthcare facilities are not reimbursed for care and services that are not medically necessary in an acute care facility when these services and care can be effectively and appropriately provided in a different setting such as a subacute care facility.

Choice A is incorrect. The principle of retrospective reimbursement is not employed when you appropriately transfer a client from an acute care healthcare facility to a subacute healthcare facility along the continuum of care because prospective rather than retrospective reimbursement is used in contemporary healthcare facilities ( Eg: Retail clinics, Assisted living facilities).

Choice B is incorrect. The principle of rehabilitation care is not employed when you appropriately transfer a client from an acute care healthcare facility to a subacute healthcare facility along the continuum of care because rehabilitation is not a principle associated with reimbursement.

Choice C is incorrect. The need to limit lengths of stay is a principle of reimbursement; however, the appropriate transfer of a client from an acute care healthcare facility to a subacute healthcare facility along the continuum of care is not based on the need to limit client length of stay but instead, the client needs.
Reference:
Sommer, Johnson, Roberts, Redding, and Churchill. Nursing Leadership and Management: Review Module Edition 6.0; ATI Nursing Education.

NCSBN Client Need: Topic: Management of Care; Sub-Topic: Establishing Priorities

42
Q

Which of the “5 Rights of Delegation”, if any, is violated and not followed when the registered nurse assigns a licensed practical nurse to perform an ongoing assessment of a client after admission?

A. The “right” circumstances.

B. The “right” supervision.

C. The “right” directions.

D. The “right” task.

A

Explanation

Correct Answer is D: The “Right of Delegation” that is violated and not followed when the registered nurse assigns a licensed practical nurse to perform an ongoing assessment of a client after admission is the “right” task.

The “Five Rights of Delegation” are:

The “right” task
The “right” circumstances
The “right” person
The “right” directions and communication
The “right” supervision and evaluation

Assessments are NOT within the practice scope for licensed practical nurses; however, licensed practical nurses can collect data for evaluations.

Choice A is incorrect. The “right” circumstances include things like the acuity of the client and time constraints and not ensuring that delegated tasks are within the staff member’s scope of practice.

Choice B is incorrect. The “right” supervision means that the nurse must supervise delegated tasks in an ongoing manner and not ensure that delegated responsibilities are within the staff member’s scope of practice.

Choice C is incorrect. The “right” directions include giving the staff member details of the delegated assignment and not ensuring that delegated tasks are within the staff member’s scope of practice.

Delegation Concepts are frequently tested on NCLEX. Master them through this high-yield video: 17 minutes video.

Reference: Berman, Audrey, Snyder, Shirlee, and Geralyn Frandsen. Kozier and Erb’s Fundamentals of Nursing: Concepts, Process, and Practice.

43
Q

The nurse is caring for a 13-year-old male child in the pediatric unit with a left below the knee cast. The boy reports pain and numbness of the foot. The nurse notes that the toes of the left foot are cold. Which of the following actions should the nurse take first?

A. Remove the cast.

B. Have the child ambulate.

C. Notify the physician.

D. Elevate the leg on two pillows.

A

Explanation

Choice C is correct. The client is already showing the signs of compartment syndrome. The client has pain, numbness, and cold feet (low perfusion). Pain, pulselessness, pallor, paresthesias, and paralysis are 5P’s associated with compartment syndrome. Compartment syndrome is an emergency. The nurse should be able to recognize signs and symptoms of compartment syndrome and notify the physician STAT.

Compartment syndrome often results after trauma and is more common in the anterior compartment of the leg. Following a trauma, there may be decreased intracompartmental space or increased intracompartmental fluid volume (due to fracture, hematoma etc.). Because the surrounding fascia is noncompliant, the compartment pressure increases. In normal circumstances, there is a balance between the venous outflow and arterial inflow. But increasing compartmental pressure results in a reduction of venous outflow. Consequently, venous pressure increases, further fueling an increase in compartmental pressure. Once compartmental pressure increases more than arterial pressure, arterial blood flow gets affected and ischemia ensues. If ischemia lasts longer, irreversible necrosis/ death of the tissue occurs.

Choice D is incorrect. After notifying the physician, the affected limb should be placed at the level of the heart, not above the heart level. While elevation above the heart level may help venous drainage, it also reduces arterial inflow further and worsens the ischemia. Please note that elevating the limb above the heart level is indicated to reduce edema and prevent compartment syndrome soon after the cast is applied. Once compartment syndrome has already happened, one must not elevate the limb.

Choice A is incorrect. The child is displaying signs of neurovascular compromise due to compartment syndrome. The cast should be removed to relieve pressure; however, it is not the first action to be taken by the nurse. Cast removal should be arranged after informing the physician. Fasciotomy may be needed, and the physician needs to know immediately.

Choice B is incorrect. The child should not ambulate as this will increase the child’s risk of further injury.
Reference
Pillitteri, A. Maternal and Child Health Nursing: Care of the Childbearing and Childbearing Family.

44
Q

Which of the following tasks would be inappropriate for an LPN/LVN to accept as an assignment?

A. Obtaining an occult blood sample from a 15-year-old client with ulcerative colitis

B. Assessing a 35-year-old client newly admitted for chest pain

C. Reinforcing teaching to a 25-year-old first-time mother regarding care of her new baby

D. Adjustment of a 69-year-old patient’s cervical traction as ordered by the attending doctor

A

Explanation

Choice B is correct. While LPNs can perform focused assessments, a registered nurse or attending physician should always perform initial assessments.

Choices A, C, and D are incorrect.Reinforcing teaching, obtaining stool samples and following orders to adjust cervical traction ( as ordered by provider) are within the scope of practice of an LPN/LVN. LPNS can not do initial teaching, but repeating or reinforcing is allowed.

NCSBN Client Need

Topic: Safe and Effective Care Environment

Subtopic: Coordinated Care

Resource: Fundamentals of Nursing (Taylor/Lillis/Lynn)

Chapter 7: Legal dimensions of Nursing Practice

Lesson: Delegated Nursing Care

45
Q

While working in an adult ICU, an 82-year old CHF patient is put on life-support. His family is informed that he is not expected to live much longer, and are asked about their wishes. They tell the team that their family practices Judaism, and they wish to follow their religious beliefs surrounding the end of one’s life. As a culturally competent nurse, you know this will include which of the following wishes? Select all that apply.

A. They do not wish for the patient to be taken off life-support.

B. The head should be kept elevated above the body at all times.

C. They would not like an autopsy to be performed.

D. They do not want any pain medication given to the patient.

A

Explanation

The correct answers are A and C. Different religions have very specific beliefs about the end of life, and therefore their wishes will vary accordingly. In Judaism, prolongation of life is very important, and the family will not want their loved one taken off of life support. The religion also prohibits autopsy and cremation, so the family will request no autopsy be performed after the death of their loved one.

B is incorrect. This is a belief of individuals following the faith of Islam.

D is incorrect. This is a belief of individuals following the faith of Buddhism.

NCSBN client need:

Topic: Psychosocial Integrity Subtopic: End-of-life care

Reference:

Silvestri, L.; Saunders Comprehensive Review for the NCLEX-RN Examination, ed 6, St. Louis, 2014, Elsevier, p. 106

46
Q

While preparing to change the dressing of a female patient with end-stage renal disease, the nurse notices that the patient’s son is silently holding her hand and praying. Which of the following should be the nurse’s initial action?

A. Continue preparing for the procedure in the room.

B. Notify the chaplain.

C. Leave the room quietly and come back after 15 minutes to change the client’s dressing.

D. Ask the son if he wants the nurse to join in prayer.

A

Explanation

Choice C is correct. The nurse should respect the client and her son in their moment of prayer and should not impose on them. The nurse’s best action is to leave the room and come back when they are finished praying.
Choices A, B, and D are incorrect.
It is inappropriate for the nurse to continue preparing for the procedure (Choice A). The nurse should respect the client and her son’s need for privacy during the prayer. The most appropriate action of the nurse is to leave the room momentarily. Unless requested by the client, the nurse should not inform the chaplain ( Choice B) or any other person. Asking the son if she can be allowed to join the prayer ( Choice D) is inappropriate. The nurse should respect their right to privacy and should not impose on the client.
Reference
Ignatavicius DD, Workman LM. Medical-Surgical Nursing: Patient-Centered Collaborative Care,

47
Q

Which of the following are components of the definition of critical thinking? Select all that apply.

A. Reasoned thinking

B. Openness to alternatives

C. Adherence to established guidelines

D. Ability to reflect

E. Loyalty to traditional approaches

F. Desire to seek truth

A

Explanation

The correct answers are A, B, D and F. Critical thinking is a combination of reasoned thought, openness to alternatives, and ability to reflect, and a desire to seek the truth.

There are many definitions of critical thinking. It is a complex concept, and people think about it in different ways. Any situation that requires critical thinking is likely to have more than one “right” answer. You do not need critical thinking to add 2 + 2 and come up with the solution. However, you do need critical thinking to problem-solve essential decisions. A crucial aspect of critical thinking is the process of identifying and checking your assumption. This is also a necessary part of the research process. Critical thinking is a combination of reasoned thought, openness to alternatives, and ability to reflect, and a desire to seek the truth.

Choices C and E are incorrect.

o C- Adhering to established guidelines does not require critical thinking.

o E- Loyalty to traditional approaches does not demonstrate critical thinking and could actually hinder it.
NCSBN Client Need
Topic: Safe & Effective Care Environment; Subtopic: Management of Care
Reference:
Fundamentals of Nursing/ Theories, Concepts, and Applications (Wilkinson/Treas/Barnett/Smith); Chapter02: Critical Thinking and Nursing Process; Lesson: What Is Critical Thinking?

48
Q

Which term best describes the nurse’s role as the nurse actively upholds and protects the rights of individual clients and groups of clients?

A. Deontological ethical practice.

B. Advocacy.

C. Utilitarian ethical practice.

D. Autonomy.

A

Explanation

The correct answer is B. The term that best describes the role of the nurse as the nurse actively upholds and protects the rights of individual clients and a group of clients is advocacy.

Choice A is incorrect. Deontological ethics is a school of ethical thought, and it does not relate to the rights of individual clients and groups of clients.

Choice C is incorrect. Utilitarian ethics is a school of ethical thought, and it does not relate to the rights of individual clients and groups of clients.

Choice D is incorrect. Autonomy is defined as the individual’s right to make independent, informed decisions without any coercion. It does not reflect the nurse’s upholding and protecting the rights of individual clients and groups of clients.
Reference:
Berman, Audrey, Snyder, Shirlee, and Geralyn Frandsen. Kozier and Erb’s Fundamentals of Nursing: Concepts, Process, and Practice.

49
Q

The nurse is educating her client about the “Do Not Resuscitate (DNR)” option. Which of the following statements is not true regarding advance directives? Select all that apply

A. One physician must determine when a patient is unable to make medical decisions for himself.

B. Advance directives must be reviewed and re-signed every ten years to remain valid.

C. Emergency Medical Technicians (EMTs) cannot honor advance directives unless a doctor has signed them.

D. An advance directive is legally valid in every state, no matter in which state it was initially created.

A

Explanation

Choices A, B, and D are correct. These statements are “Not True.”

Once a patient arrives at the hospital, physicians will need to evaluate the patient and implement the advance directive, if necessary. Two physicians, not one (Choice A), should determine whether a patient is unable to make decisions for himself.
Advance directives do not expire and remain in effect until they are changed. It is not true that they need to be signed every ten years to stay valid (Choice B).
Some states do not honor advance directives created in other countries. So, if a patient moves, he/she should check with his/her new state policies on the topic (Choice D).

Choice C is incorrect. Advance Directives cannot be honored by Emergency Medical Technicians (EMTs) unless a doctor has signed them. If summoned, EMTs are legally obligated to provide emergency care to a patient regardless of advance directive status unless a physician has signed the directive. Choice C is a TRUE statement and, therefore, an incorrect option here.
NCSBN Client Need
Topic: Safe and Effective Care Environment; Subtopic: Coordinated Care

Reference: Fundamentals of Nursing (Taylor/Lillis/Lynn); Chapter 29: Perioperative Nursing; Lesson: Informed Consent and Advance Directives

50
Q

The nurse is preparing to sign a patient’s surgical consent form after the physician has explained the procedure to the patient and family. As the patient signs the form, she comments “I really didn’t understand most of what the doctor said, but I have to have this procedure, so I want to sign.” Which is the appropriate nursing action?

A. Witness the document, as the patient states she wants to sign it.

B. Notify the physician or nursing supervisor.

C. Call the OR to cancel the procedure and reschedule at a later date.

D. Explain the information she did not understand.

A

Explanation

Correct Answer B. The person ( here, the doctor) responsible for performing the procedure has the responsibility to obtain the patient’s consent, providing a clear explanation about the procedure and all associated risks. When witnessing the patient’s signature, the nurse confirms that the patient understands the information about the procedure. If the patient denies understanding, the nurse must contact the physician or the nursing supervisor.

Choice A is incorrect. The nurse has to witness the patient’s signature but even prior to that, she must confirm that the patient understood the information about the procedure.

Choice C is incorrect. The nurse must call the physician or nursing supervisor and inform that the patient did not understand the procedure information. Canceling the procedure is not necessary as something else needs to be done right now.

Choice D is incorrect. It is the responsibility of the person performing the procedure ( here, the doctor) to obtain the patient’s consent, providing a clear explanation about the procedure and all associated risks. The nurse only needs to confirm if the patient understood it.
NCBSN Client Need:
Category: Management of care; Sub-topic: Informed Consent.
Reference:
Potter, P., Perry, A., Stockert, P., Hall, A., Fundamentals of Nursing 8th Edition. Elsevier Mosby St Louis 2013.

51
Q

Which of the following is a critical and necessary component of a malpractice case?

A. An act of omission.

B. An act of commission.

C. An intentional act.

D. A breach of duty.

A

Explanation

The Correct Answer is D.A breach of duty is a critical and necessary component of a malpractice case. Other vital and essential elements of a malpractice case include an act of commission or omission, an intentional or unintentional act, damages to the client, causation, foreseeability, and causation.

Choice A is incorrect. A malpractice case can occur as the result of both acts of omission and commission. Therefore, the commission is not a necessary component of a malpractice case.

Choice B is incorrect. A malpractice case can occur as the result of both acts of omission and commission. Therefore, failure is not a necessary component of a malpractice case.

Choice C is incorrect. A malpractice case can occur due to intentional and negligent, or unintentional acts; therefore, the intention is not a necessary component of a malpractice case.

Reference: Berman, Audrey, Snyder, Shirlee, and Geralyn Frandsen. Kozier and Erb’s Fundamentals of Nursing: Concepts, Process, and Practice.

52
Q

ich of the following is a priority that must be considered and critically thought about by the nurse before referring a client to a healthcare setting or service external to their current healthcare setting?

A. The external healthcare setting’s or service’s cultural values and beliefs.

B. The external healthcare setting’s or service’s admission criteria.

C. The current healthcare facility’s actual and potential census.

D. The current healthcare facility’s actual and potential case mix.

A

Explanation

The correct answer is B. The external healthcare setting’s or service’s admission criteria is the priority that must be considered and critically thought about by the nurse before referring a client to a healthcare setting or service external to their current healthcare setting.

Choice A is incorrect. Although the external healthcare setting’s or service’s cultural values and beliefs should be considered, it is not the priority that must be found and critically thought about by the nurse before referring a client to a healthcare setting or service external to their current healthcare setting.

Choice C and D are incorrect. The current healthcare facility’s actual and potential census is not a consideration that the nurse should think about before referring a client to a healthcare setting or service external to their current healthcare setting; it is the client’s needs that must be considered.

Reference: Sommer, Johnson, Roberts, Redding, and Churchill. Nursing Leadership and Management: Review Module Edition 6.0; ATI Nursing Education.

53
Q

You are supervising a new Licensed Practical Nurse (LPN). You should let her know which of the following is inappropriate with regard to delegating a task to an Unlicensed Assistive Personnel (UAP)? Select All That Apply.

A. Delegate the task to the aide, watch her perform the task without her seeing you, and follow up to ensure the task was done correctly.

B. Delegate the task to the nurse aide, confirm her understanding, and follow up to ensure the task was done safely.

C. Delegate the task to the nurse aide, supervise if needed, and check-in after the task is complete to see if help is required.

D. Delegate the task to the nurse aide, ensure understanding of the job, and supervise the task being performed.

A

Explanation

Choices A, B, and D are correct. These are inappropriate actions while delegating a task. Delegation is transferring responsibility for a task. However, accountability for the task being performed is shared. It is the delegator’s responsibility to make sure the delegatee understands the job before handing off the burden of showing it, not after delegating (Choice B).

Supervision should be available on-site, if necessary. Specific direction by the nurse to UAP when assisting the nurse with a task and under the direct visual supervision of the nurse is not considered delegation (Choices A and D). Delegator is also responsible to follow up after the task has been completed to make sure it was done safely and correctly.

Choice C is incorrect. This is the only listed option that reflects the appropriate action when delegating an assignment to a nursing assistant.
NCSBN Client Need
Topic: Safe and Effective Care Environment;Subtopic: Coordinated Care

Resource: Fundamentals of Nursing (Taylor/Lillis/Lynn);Chapter 14: Implementing;Lesson: Delegating Nursing Care

54
Q

The nurse walks in to the room and finds her client complaining of severe shortness of breath and chestpain. She suspects Pulmonary Embolism. After notifying the rapid response team, the nurse’s priority action is:

A. Obtain vital signs and place the client in left-sided, Trendelenburg position.

B. Administer heparin.

C. Check lung sounds.

D. Elevate the head of the bed.

A

Explanation

Choice D is correct. The first action following the notification of the rapid response team when a pulmonary embolism is suspected is “elevating the head of the bed” to about 30 degrees. This is a quick action that does not require a doctor’s order.

Pulmonary embolism causes ventilation and perfusion mismatch. In a position with head of the bed elevated, gravity pulls the diaphragm downward, allowing for lung expansion and improved ventilation.

Please note that an embolus may refer to a blood clot (pulmonary embolism, arterial thromboembolism), air bubble (air embolism), or a piece of fatty deposit (fat embolism) that can be carried into bloodstream to lodge in a vessel and cause an embolism. Many students make a knee jerk selection of Trendelenburg position the moment they see the word embolism in the question. Please note that Trendelenburg position or left lateral position is used in patients with “air” embolism. This is because air is a gas and it will float in the upper part of the right ventricle/ right atrium when patients are placed in such a position.Pulmonary embolism (PE) is a blood clot. Often, it travels from lower extremities to lungs. You do not want to keep a patient with acute PE in aTrendelenburg position because that may facilitate further embolism in an acute thromboembolism scenario.

Choice A is incorrect. The patient is short of breath and is clearly in distress. Vital signs should be taken after the patient’s head of the bed is elevated, and oxygen has been initiated.

Choice B is incorrect. While this patient may receive heparin, a doctor’s order will be needed to initiate heparin.

Choice C is incorrect. The patient is in distress. Rapid response needs to be notified and the head end of the bed needs to be elevated prior to proceeding with further assessment. The nurse may assess lung sounds after the head of the bed is elevated, and oxygen has been initiated.

NCSBN client need | Topic: Physiological Integrity, Reduction of Risk Potential
Reference:
Sole M, Klein D, Moseley M. Introduction To Critical Care Nursing. 1st ed. St. Louis, Mo.

55
Q

The nurse is providing care for a patient recently transferred from the post-anesthesia care unit [PACU]. The chart indicates that the patient was medicated for pain 1 hour ago, yet the patient reports that he is experiencing extreme pain. He is not due for further medication for another 2 hours. How might the nurse intervene in the nursing role of a patient advocate?

A. Contact the physician regarding the need for more effective pain management.

B. Assist the patient to use non-pharmacological pain management strategies.

C. Explain to the patient that giving the pain medication too soon can be dangerous.

D. Provide a quiet environment to help the patient rest and cope with his pain level.

A

Explanation

Correct Answer is A. an essential aspect of advocacy is speaking on behalf of the patient, to help meet the patient’s needs, such as when calling the physician to discuss the need for more effective pain management – based in this case on the patient’s fundamental right to be free from pain.

Choices B and D are incorrect. These are nursing interventions that can be employed to enhance the prescribed pain medication but do not meet defining characteristics related to advocacy.

Choice C is incorrect. While this is factual information, it does not address the need to provide adequate pain management.

Bloom’s Taxonomy – Analyzing
Reference:
Hyland, D., 2002. An Exploration of the Relationship Between Patient Autonomy and Patient Advocacy: Implications for Nursing Practice. Nursing Ethics 9(5)

56
Q

A 9-year-old child diagnosed with leukemia is scheduled for a bone marrow aspiration tomorrow. Regarding his informed consent, which initial nursing action is most appropriate?

A. Obtain assent from the child.

B. Have his parents sign the consent.

C. Have the physician sign the consent.

D. Witness the informed consent

A

Explanation

A is correct. The child needs to have some control and input in the decision making process regarding his care. Assent means the child has been fully informed about the procedure and concurs with those giving the informed consent.

B is incorrect. A minor is a person under 18 years of age, not married and has not been married, or has not had the disabilities of minority removed by the court. Since the child is under 18, the parents must sign the informed consent form. However, the initial action should be to obtain a child’s assent. Both the parents and the nurse must first obtain permission from the child.

C is incorrect. The physician cannot sign informed consent for the minor child. Legally, informed consent for a minor can be signed by the natural mother/ father; adoptive mother/father, a parent who is appointed managing conservator (even for invasive procedures), and a parentwho is appointed possessory conservator (as long as not for invasive procedures). In the absence of the above persons to sign the consent, a grandparent, adult sister/ adult brother, or an educational institution who has possession of the minor child can sign the informed consent.

D is incorrect. The nurse can witness the signing of the informed consent for the procedure. However, it is not the priority action for the situation.
Reference
Pillitteri, A. Maternal and Child Health Nursing: Care of the Childbearing and Childbearing Family, 4th Edition.

57
Q

ou have been asked to witness informedconsent with your signature and title. What does this signature signify and mean?

A. It means that you have taught the client about the procedure or treatment.

B. It means that you can validate the client’s signature and identity.

C. It means that you can confirm and validate that the client is fully informed.

D. It means that you have taught the client, and you can confirm their knowledge.

A

Explanation

The correct answer is B. When you, as a nurse, witness an informed consent with your signature and title, this signature signifies and means that you can validate the client’s name and identity. It does not say that you taught the client about the procedure or treatment or confirm their understanding of the process or treatment.

Choice A is incorrect. When you, as a nurse, witness an informed consent with your signature and title, this signature does NOT signify or mean that you have taught the client about the procedure or treatment. It means something else.

Choice C is incorrect. When you, as a nurse, witness an informed consent with your signature and title, this signature does NOT signify or mean that you can confirm and validate that the client is fully informed about the procedure or treatment. It means something else.

Choice D is incorrect. When you, as a nurse, witness an informed consent with your signature and title, this signature does NOT signify or mean that you have taught the client, and you can confirm their knowledge about the procedure or treatment. It means something else.

Reference: Berman, Audrey, Snyder, Shirlee, and Geralyn Frandsen. Kozier and Erb’s Fundamentals of Nursing: Concepts, Process, and Practice.

58
Q

You overhear a nursing assistant tell a client that they will have to get a feeding tube if they do not start eating more at mealtimes. What has this nursing assistant do? Select all that apply.

A. The nursing assistant has committed a battery.

B. The nursing assistant has emotionally abused the client.

C. The nursing assistant has committed assault.

D. The nursing assistant has been negligent.

A

Explanation

Correct Answer is B and C. The nursing assistant has committed assault, and they have also emotionally abused the client. Charge like emotional abuse occurs when someone causes another person, like a client, to feel fearful and threatened.

Choice A is incorrect. The nursing assistant has not committed battery; the battery is the actual wrongful and inappropriate touching of a client or an intentional act to do physical harm.

Choice D is incorrect. The nursing assistant has not been negligent. Negligence is failing to do something that should have been done according to established standards of care and practice.
References:
Berman, Audrey, Snyder, Shirlee, and Geralyn Frandsen. (2016). Kozier and Erb’s Fundamentals of Nursing: Concepts, Process, and Practice (10th Edition)

59
Q

The charge nurse is planning patient care assignments for a licensed practical/vocational nurse (LPN/VN). Which of the following would be an appropriate patient assignment for the LPN? Select all that apply.

A. 67-year-old one-hour post-procedure from a cardiac catheterization.

B. 88-year-old client just admitted for intractable pain secondary to metastatic cancer.

C. 42-year-old being discharged following a diagnosis of diabetes mellitus type II.

D. 75-year-old client inpatient with colon cancer needing colostomy care.

E. 50-year-old client being treated for herpes zoster with prescribed oral antivirals

A

Explanation

Correct Answers, D, and E.
Scenarios D and E are within the scope of LPN practice. LPNs can be assigned Ostomy care and may administer oral medications to stable clients.
Client in Scenario C -is being discharged and requires Discharge teaching. LPNs can not be assigned initial discharge teaching, although they can reinforce patient education.
Client in Scenario A requires close monitoring and likely to be considered unstable and needs RN. LPNs should be assigned patients who are predictable/stable but NOT immediate post-operative patients.
The client in Scenario B is just admitted and is unpredictable from pain. LPN can not be assigned a new admission.

A licensed practical nurse (LPN) can monitor and attend “stable” patients as long as they have been evaluated by a registered nurse (RN). As such an LPN can

Administer oral and parenteral medications. LPN may not administer IV medications in individual states. In NCLEX, a student is better off not choosing that option.
LPN may reinforce patient teaching/ education that was initially addressed by a Registered Nurse
LPN may perform focused assessments but only after the RN's initial estimate. The performance of the first assessment is outside the scope of the LPN.
LPN may attend routine procedures like bladder-catheterization, address tube feeding and ostomy care

NCSBN Client need
Topic: Management of care; Sub-Topic: Assignment, delegation, and supervision.

60
Q

Which of these critical thinking and supervision skills is necessary for effective and appropriate supervision after the nurse assigns and delegates tasks to staff members?

A. The ongoing observation and determination of staff’s time management skills.

B. The ongoing one-to-one observation and staff competency validation and documentation.

C. The observation and determination of staff’s time management skills at the end of the shift.

D. The one-to-one observation and staff competency validation and documentation at the end of the shift.

A

Explanation

The correct answer is A.The critical thinking and supervision skills necessary for effective and appropriate supervision after the nurse assigns and delegates tasks to staff members are the ongoing observation and determination of staff’s time management skills to ensure that complete care is given to the clients before the end of the shift.

Choice B is incorrect. Ongoing one-to-one observation and staff competency validation and documentation are not routinely necessary for effective and appropriate supervision after the nurse assigns and delegates tasks to staff members. Still, a determination of the completion of client care is.

Choice C is incorrect. The observation and determination of the staff’s time management skills at the end of the shift are too late. This supervision is an ongoing process to ensure that all client care is provided promptly.

Choice D is incorrect: The one-to-one observation and staff competency validation and documentation at the end of the shift is not routinely necessary for the effective and appropriate supervision after the nurse assigns and delegates tasks to staff members, but a determination of the completion of client care is

Reference: Berman, Audrey, Snyder, Shirlee, and Geralyn Frandsen. Kozier and Erb’s Fundamentals of Nursing: Concepts, Process, and Practice.

61
Q

The nurse is about to lift a 350-pound patient using an electric lift from the bed and transfer him to a stretcher. What should be the priority nursing action?

A. Call for assistance from two staff members.

B. Make sure the client is correctly positioned in the lift prior to lifting.

C. Slowly lift the client off the bed.

D. Make sure the stretcher is locked.

A

Explanation

Choice B is correct. The safety of the client should take priority. The nurse must ensure that the client is safely secured and adequately attached to the lift. Incorrect positioning of the client in the lift’s sleeves might put the client at risk for falls.

Choice A is incorrect. The lift can be done by two persons, the nurse and one other staff; there is no need to call for two staff members. Moreover, the priority action is to ensure safety by securing the patient to the lift and ensuring proper positioning.

Choices C and D are incorrect. The nurse should ensure that the stretcher is locked and also slowly lift the client. However, the priority action is first to make sure the client is correctly positioned.
Reference
Ignatavicius DD, Workman LM. Medical-Surgical Nursing: Patient-Centered Collaborative Care, 7th ed. St. Louis, MO: Elsevier

62
Q

A client in the post-anesthesia care unit is semiconscious and dyspneic. He exhibits retraction of intercostal muscles, and his oxygen saturation is 88%. What is the nurse’s priority action at this time?

A. Place a pillow under the client’s head

B. Insert an oropharyngeal airway

C. Administer oxygen by mask

D. Reposition the client in a side-lying position.

A

Explanation

Choice D is correct. The priority action in the care of a post-anesthesia client is to secure a patent airway. The most common cause of airway obstruction in a semiconscious or unconscious patient is the tongue. The anesthetic agents and muscle relaxants used during surgery may cause relaxation of tongue and jaw muscles, causing posterior movement of the language and epiglottis, which leads to the obstruction of the airway.

The client is dyspneic, using intercostal muscles and manifesting symptoms of airway obstruction. Repositioning in the side-lying position with the face slightly down is a simple initial nursing action that can prevent occlusion of the pharynx and allow the drainage of mucus from the mouth.

If repositioning fails to improve the condition, other airway positioning maneuvers (for example, head-tilt-chin lift, jaw-thrust) should be attempted. Supplemental oxygen (Choice C) can be administered once the airway patency is established. If these initial interventions fail to maintain a patent airway, then an airway adjunct (Oropharyngeal or Nasopharyngeal airway) should be used (Choice B).

Choice A is incorrect. A pillow under the head increases the risk of aspiration or airway obstruction.

Choice B is incorrect. Because the issue is airway obstruction, efforts to promote an open airway are most appropriate. The first step, however, is to open the airway via non-invasive measures. A simple initial intervention, such as repositioning, may help by making the tongue move forward. If those initial non-invasive measures fail, move to insert an airway adjunct. Additionally, because this client is semiconscious, a nasopharyngeal airway would be appropriate if the initial interventions failed.

If the patient is semi-conscious and can cough, they still have a gag reflex, and an oral airway is contraindicated. An oropharyngeal (Guedel) airway helps to secure a patent airway by preventing the tongue from blocking the epiglottis. However, because of the depth of an adequately inserted oropharyngeal airway, it can only be used in an “unconscious” client with no gag reflex.

Choice C is incorrect. The issue is airway obstruction, not the percentage of available oxygen (Fio2). Providing supplemental oxygenation without addressing the airway obstruction is futile.

NCSBN Client Need

Topic: Physiological Integrity; Subtopic: Physiological Adaptation

Resource: Kozier and Erb’s Fundamentals of Nursing; Chapter 37: Perioperative Nursing; Lesson: Postoperative Phase

63
Q

The nurse is discussing information about advanced directives with a patient who expresses concerns, asking, “What if I change my mind about what I want?” What approach would you use to respond to the patient’s care?

A. Explain that the patient would have to file a new witnessed document in order to make any changes.

B. Discuss the need to be very sure about his preferences, as the living will is a binding legal document.

C. Assure the patient that he can change or revoke his advanced directives at any time.

D. Advise the patient that changes could not be made during this hospital stay.

A

Explanation

Correct Answer is C. The patient may revoke either a living will or durable power of attorney at any time, and this can be done either verbally or in writing.

Choice A is incorrect. The patient can revoke an advanced directive verbally; a newly written document is not required.

Choices B and D are incorrect. While the living will is a legal document, advanced directives can be easily changed at any time, just by saying so verbally, or with a newly written report.

Blooms Taxonomy – Applying
Reference:
Potter, P., Perry, A., Stockert, P., Hall, A., Fundamentals of Nursing 8thEdition. Elsevier Mosby St Louis 2013.

64
Q

The client’s nephew walks up to the nurse’s station and asks if he can see his uncle’s file. The nephew states, “It’s okay, I’m a nurse as well. I just want to take a quick look and see how my uncle is doing.” What is the nurse’s most appropriate response?

A. “You can take a look for only 5 minutes.”

B. “Let me get an approval from the attending physician.”

C. “I will need permission from your uncle first.”

D. “Non-hospital employees can not view the patient’s file.”

A

Explanation

The correct answer is C. According to the Health Insurance Portability and Accountability Act (HIPAA), the relative must first obtain consent from the client to view his file.

Choice A is incorrect. According to the Health Insurance Portability and Accountability Act, the relative must first obtain consent from the client to view his file. In the absence of the client’s permission, allowing the nephew to view the data even for 5 minutes is not legal.

Choice B is incorrect. The physician is not the one that decides who can view the client’s file. The client’s consent is necessary under HIPAA provisions.

Choice D is incorrect. Non-employees can view the client’s file once the client has given consent for them to see his data.
Reference
Potter, PA, Perry. AG, Stockert, PA, Hall, AM. Fundamentals of Nursing, 8th ed. St. Louis, MO: Elsevier Mosby;2013

65
Q

When making patient care assignments, the nurse recognizes that the following tasks are within the scope of practice for nursing assistive personnel [NAP], with which exception?

A. Providing catheter care.

B. Performing range of motion exercises.

C. Changing the colostomy skin barrier.

D. Encouraging the use of self-help devices.

A

Explanation

Correct Answer is C. The NAP scope of practice allows the NAP to empty ostomy bags or change bags that do not adhere to the skin; the NAP could apply a new pack to a two-piece system, but not the adhesive skin barrier component.

Choices A, B, and D are incorrect. Providing catheter care, performing a range of motion exercises, and encouraging the patient to use self-help devices are within the NAP scope of practice.

Bloom’s Taxonomy: Analyzing
Reference:
McMullen, Tara L. et al. Certified Nurse Aide Scope of Practice: State-by-State Differences in Allowable Delegated Activities Journal of the American Medical Directors Association, 2015; 16(1) 20 - 24

66
Q

After reviewing information related to advanced directives with a patient, which statement by the patient indicates the need for further discussion and education?

A. “A living will designates a person that can make decisions about my medical care if I can’t do that myself.”

B. “The person who I choose to make decisions about my medical care if I can’t is named on the durable power of attorney.”

C. “I can refuse to be intubated or placed on mechanical ventilation as part of a living will.”

D. “If I change my mind, I can revoke an advanced directive any time, just by verbally saying so.

A

Explanation

The correct answer is A. This statement indicates that the patient requires further teaching, as a living will provide specific instructions to health care providers regarding the patient’s preferences about life-sustaining interventions, eg: Cardiopulmonary resuscitation, mechanical ventilation, dialysis, tube feeding, organ and tissue donations, body donation and comfort care.
An advance directive in which a person is designated to make decisions for the patient when he/she is unable to do so is called a durable power of attorney or a healthcare proxy and is not a part of a living will.

Choice B is incorrect. This statement indicates the patient understands the purpose of a durable power of attorney. A living will differ from the durable power of attorney for health care because life will delineate the patient’s wishes precisely. In contrast, a power of attorney allows the patient’s designated agent to make health care decisions for the patient.

Choice C is incorrect. This statement indicates the patient understands the purpose of the living will.

Choice D is incorrect. This statement indicates that the patient understands that he can revoke an advanced directive any time and that this can be done either verbally or in writing.

Bloom’s Taxonomy – Analyzing
Reference:
Potter, P., Perry, A., Stockert, P., Hall, A., Fundamentals of Nursing 8thEdition. Elsevier Mosby St Louis 2013.

67
Q

The nurse notes that the physician has entered a do not resuscitate [DNR] order. However, there is no advanced directive by the patient present on the patient’s chart. Which is the appropriate nursing action?

A. Notify the physician about the need for a living will to validate this order.

B. Verify that the physician consulted with the patient and/or family.

C. Accept the order as written, no other documentation is needed.

D. Notify the nurse supervisor and risk management about the DNR order.

A

Explanation

The correct answer is B. Documentation that the physician has consulted with the patient and family is required before a do not resuscitate order is entered on the patient’s chart.

Choice A is incorrect. It is not necessary to have a living will on the patient’s chart, but there must be documentation that the issue was discussed with the patient/family.

Choice C is incorrect. There must be documentation noting that the DNR order was discussed with the patient and family.

Choice D is incorrect. It is not necessary to notify the nursing supervisor and risk management about this order.

Bloom’s Taxonomy – Applying
Reference:
Potter, P., Perry, A., Stockert, P., Hall, A., Fundamentals of Nursing 8thEdition. Elsevier Mosby St Louis 2013

68
Q

The nurse cares for an infant undergoing a surgical repair of a total anomalous pulmonary venous return tomorrow. The doctor has talked to the parents and obtained consent. The mother tells the nurse, “ I’m not so sure about this. What if my baby dies?”. The nurse’s most appropriate response is:

A. Explain the procedure to the mother

B. Notify the surgical team and have them come back to speak with the mother.

C. Reassure the mother that everything will go as planned.

D. Tell the mother that because she has already signed the consent, she cannot change her mind now.

A

Explanation

Choice B is correct. The nurse has identified that the mother has concerns about the surgery, so it is her responsibility to notify the surgical team and have them come back to speak with the mother.

Choice A is incorrect. It is not the responsibility of the nurse to explain the surgical procedure to the mother. This would be acting outside of her scope of practice and would not be appropriate. The surgeon/ surgical team doing the procedure should be the one explaining it again to the mother.

Choice C is incorrect. It is not appropriate to reassure the mother that everything will go as planned. There are always risks involved with the surgery, and it would be inappropriate to make such statements.

Choice D is incorrect. It is not appropriate to tell the mother that she cannot change her mind because she has already signed the consent paperwork. The child’s legal guardian does have the ability to change their mind and should not be discouraged from asking questions.

NCSBN Client Need: Topic: Psychosocial Integrity

69
Q

During nursing school, you have been taught to be open and frank with all clients, even when information may be upsetting and distressing to the client. By doing this, you are following the ethical principle of:

A. Beneficence.

B. Veracity.

C. Nonmalficence.

D. Fidelity.

A

Explanation

The correct answer is B. You follow the ethical principle of integrity when you are open, frank, and truthful with all clients, even when information may be upsetting and distressing to the client.

Choice A is incorrect. You are not following the ethical principle of beneficence when you are open, frank, and truthful with all of your clients, even when information may be upsetting and distressing to the client. Beneficence is defined as “doing good” for our patients and not being frank and open with clients’ communication.

Choice C is incorrect. You are not following the ethical principle of nonmaleficence when you are open, frank, and truthful with all of your clients, even when information may be upsetting and distressing to the client. Nonmaleficence is defined as “doing no harm” to our patients.

Choice D is incorrect. You are not following the ethical principle of fidelity when you are open, frank, and truthful with all of your clients, even when information may be upsetting and distressing to the client. Fidelity is defined as being faithful to our promises to our patients.

Reference: Kozier and Erb’s Fundamentals of Nursing: Concepts, Process, and Practice.

70
Q

The nurse is caring for a patient who is to receive a blood transfusion. Which of the following option(s) is/are considered inappropriate delegation(s) to UAP?
Select All That Apply.

A. Assist the nurse to cross-check patient’s identification with blood bag patient identifiers.

B. Monitor for shortness of breath during transfusion.

C. Record all vital signs prior to infusion.

D. Request blood products from blood bank as directed by RN.

A

Explanation

Choices A, B, and D are correct.
Choice A is correct. Cross-checking must be done with two nurses, never the UAP.
Choice B is correct. Monitoring for adverse reactions of interventions falls under the scope of the nurse, not the UAP. The UAP can check vitals after the nurse has stayed with the patient and checked vitals during the first 15 minutes.
Choice D is correct. The nurse should request the blood products from the blood bank; it would not be appropriate to delegate this task. The UAP may obtain the blood products when ready but would not be able to request.
Choice C is incorrect.It would be appropriate for the UAP to check and record this patient’s vital signs before the RN administering the blood.
NCSBN Client Need:
Topic :Leadership/management;Sub-topic:Delegation

Reference:(Lewis, Dirksen, Heitkemper, Bucher, & Camera, 2011, p. 707)

71
Q

The nurse is obtaining consent for surgery from a client. What should be the initial action of the nurse while obtaining consent?

A. Determine if the client has sufficient knowledge about the procedure.

B. Witness the signature of the client.

C. Tell the client that obtaining a signature is routine prior to surgery.

D. Explain the risks involved in the surgery.

A

Explanation

Choice A is correct. “Informed” consent means that the client must understand the procedure, the alternative options, and the risks and consequences involved. The nurse should make sure that the client has sufficient knowledge about the procedure before asking him to sign a consent.

While it is crucial for the client to know the risks of the procedure before signing the consent, those risks should be explained to the client by the doctor, not the nurse.

Choice B is incorrect. The nurse should first assess the client’s understanding of the surgery/procedure before signing as a witness.

Choice C is incorrect. Procuring the client’s signature for consent is routine before the surgery. However, just telling this to the client does not satisfy the client’s right to informed consent.

Choice D is incorrect. Explaining the procedural risks involved is not the nurse’s responsibility and should be done by the doctor.

72
Q

A client with bowel perforation is scheduled for surgery and has just been given his pre-operative medication, Diazepam. However, the new resident physician forgot to let the client sign the consent form. He hurriedly tries to make the patient sign the consent and asks the nurse to witness the signing. Which intervention by the nurse is most appropriate?

A. Witness the consent form.

B. Have the wife sign the consent for the patient and witness the signature.

C. Reschedule the surgery.

D. Report the physician to the nurse manager.

A

Explanation

The correct answer is B. Informed consent must always be obtained before pre-medication/ sedation. Unfortunately, in this case, it was missed. If the surgery is urgent, and a client is declared mentally or emotionally incompetent, the next of kin has the legal authority to give consent. In this case, the surgery is urgent (bowel perforation) and cannot be deferred, the patient is temporarily disabled, and therefore, the wife has the authority to sign the consent.

If the client is declared medically or emotionally incompetent to sign informed consent, it is essential to determine whether the loss of decision-making capacity is temporary or permanent. The next step is to determine whether the surgery is elective or urgent or an emergency. In cases where the function is temporarily lost, one should defer an elective surgery until it returns. In cases where surgery is an emergency, proceed based on the physician’s decision of the patient’s best interests. In cases where surgery is urgent, the patient’s surrogate can consent. However, if the capacity is permanently lost (e.g., dementia), the operation may proceed if felt to be in the patient’s best interest, unless the patient had previously refused the procedure before they lost capacity.

Informed consent is the process by which a patient is provided with information to make an informed decision regarding a proposed treatment and serves to educate patients benefits, risks, and potential complications of the proposed surgery. Patients have a right to be informed about the proposed treatment and determine whether they wish to move forward with the recommended treatment. It is essential that a patient has been allowed to ask questions and have them answered, understands the information, and is mentally competent to provide that consent.

There may be times when the patient does not meet one or more of the criteria for valid consent. It is essential to recognize those conditions where it may not be appropriate for the patient to provide consent and to follow suitable alternatives.

A Pre-medicated Patient: A patient pre-medicated/ sedated with a drug that can affect their judgment is one such situation. Informed consent must always be obtained prior to administering sedatives except in emergencies. When the medicated/ sedated patient does not understand the planned procedure or is not capable of making a reasonable decision, the procedure should be delayed until either the patient regains capacity or the determination is made that the procedure is urgent enough to warrant consent by a surrogate.

The Incapacitated or Incompetent Patient

    When the patient is unable to provide consent, there are several options based on the urgency of the situation.
    In an emergency, consent can be inferred or implied based on the physician's perceptions of the patient's best interests.
    In urgent and elective situations, consent may be obtained by an identified agent based on the patient's Advanced Directive and Power of Attorney for Healthcare, Durable Medical Power of Attorney, or a legally designated guardian.
    In the absence of an agent or guardian, a surrogate decision-maker may be used and may be defined in state statutes.
    The following classes of the patient's family, in descending order, may act as a surrogate in many states:
        The spouse unless legally separated
        An adult who shares an emotional, physical and financial relationship with the patient like that of a spouse
        An adult child
        A parent
        An adult brother or sister
        An adult grandchild
        An adult niece or nephew related by blood or adoption
        An adult aunt or uncle related by blood or adoption
        Another adult relative related by blood or adoption, who is familiar with the patient's personal values.
    In the absence of specific state law, the following individuals, in descending order of authority, may serve as a surrogate:
        The patient's spouse
        Adult child
        Parent
        Adult sibling
        Adult grandchild
        Any adult who has exhibited special care and concern for the patient and who is personally familiar with the patient's values.

When questions arise regarding surrogacy and absent emergent procedures, seek legal counsel in exceptional cases.

Choice A is incorrect. If the client is under the influence of chemical agents such as alcohol or drugs that impair decision-making capacity, the client is deemed mentally incompetent. Such a client’s signature does not hold ground legally.

Choice C is incorrect. Perforated bowel needs urgent surgery as it can be fatal. In the setting of needing immediate surgery, one should not reschedule the operation since the wife can give consent on behalf of the client.

Choice D is incorrect. A better alternative exists in the options where the Nurse can get the consent signed by the patient’s spouse. There is no need to report the physician right now to the nurse manager unless the physician insists that an incapacitated patient must sign the consent despite being offered a spousal alternative.

Reference

Ignatavicius DD, Workman LM.Medical-Surgical Nursing: Patient-Centered Collaborative Care, 7thed. St. Louis, MO: Elsevier; 2013

73
Q

A patient in the prenatal clinic has stated her intention to choose formula feeding for her infant. Identify which action by the nurse is most consistent with the nurse’s role of patient advocate.

A. Remind the patient of why breast feeding is the best method of infant feeding.

B. Request a referral to the lactation consultant.

C. Determine the patient’s knowledge base related to infant feeding options.

D. Accept the patient’s decision without further discussion

A

Explanation

Correct Answer C. A central concept of patient advocacy is ensuring that the patient’s decisions are based on sufficient information and understanding, supporting the patient’s right to exercise autonomy.

Choice A is incorrect. This answer does not serve to support the patient’s right to autonomy.

Choice B is incorrect. A referral to the lactation consultant is not necessarily indicated.

Choice D is incorrect. While the nurse should support the patient’s choice, it is essential to confirm that the patient’s decision-making process is based on adequate information.

Bloom’s Taxonomy – Analyzing.
Reference:
Hyland, D., 2002. An Exploration of the Relationship Between Patient Autonomy and Patient Advocacy: Implications for Nursing Practice. Nursing Ethics 9(5)

74
Q

he concept of management is most closely associated with:

A. Decision-making, problem-solving, and priority setting.

B. Inspirational abilities and coaching.

C. Visionary abilities and supervision.

D. Motivational and visionary abilities.

A

Explanation

The correct answer is A. The concept of management is most closely associated with decision-making, problem-solving, priority setting, and collaboration and coordination of patient care services.

Choice B is incorrect. Inspirational abilities and coaching are most closely associated with leadership and supervision, respectively, and not management.

Choice C is incorrect. Visionary abilities and supervision are most closely associated with leadership and supervision, respectively, and not management.

Choice D is incorrect. Motivational and visionary abilities are most closely associated with leadership and not management.

Reference: Berman, Audrey, Snyder, Shirlee, and Geralyn Frandsen. Kozier and Erb’s Fundamentals of Nursing: Concepts, Process, and Practice.

75
Q

The nurse is having her shift in the Emergency Department and is assigned to triage patients coming in for treatment. Which patient should the nurse attend to first?

A. a client complaining muscle pains, and headache

B. a client complaining of a sore foot after twisting his ankle in a game of basketball

C. a client who is manifesting shortness of breath, wheezing and cyanosis followed by a bee sting

D. a client with a splinted fractured humerus being escorted by paramedics

A

Explanation

C is correct. Upon initial presentation, the client is undergoing an anaphylactic shock. Clients with severe respiratory distress are classified as emergent and should be given priority.

A is incorrect. Triaging is a system of client evaluation to establish priorities and assign appropriate treatment and personnel. A patient complaining of muscle aches and a headache should fall under non-urgent priority.

B is incorrect. Client conditions such as sprains, minor lacerations, and cold symptoms are also classified as non-urgent.

D is incorrect. Client conditions like fractures are classified as urgent and should be given treatment as soon as possible. However, new cases always supersede that of critical situations.

Reference
Black, JM, Hawkes, JH; Medical-Surgical Nursing: Clinical Care for Positive Outcomes 8th edition, Nebraska: Elsevier